Sunteți pe pagina 1din 121

GOC FOR GMDSS REVIEWER

COMPETENCE 1 Transmit and Receive information using GMDSS subsystem and equipment and fulfilling the
functional requirements of GMDSS (GOC)
KUP1 – SEARCH AND RESCUE RDIO-COMMUNICATION, INCLUDING PROCEDURES IN THE
INTERNATIONAL AERONAUTICAL AND MARITIME SEARCH AND RESCUE (IAMSAR) MANUAL.

1. What is mean by term radio silence?


a) Stations not directly involved with the on-going Distress communications may not transmit on
the Distress frequency or channel.
b) Stations remaining off the air to safeguard proprietary information.
c) Two three-minute silent periods, at 15 and 45 minutes after the hour that provide a transmitting
"window" for distressed vessels to transmit Distress alerts using J3E.
d) Communications on a Distress frequency or channel is banned for 24 hours following the
cessation of the Distress traffic

2. How should the signal from a Search and Rescue Radar Transponder appear on a radar display?
a) A series of dashes.
b) A series of spirals all originating from the range and bearing of the SART.
c) A series of 12 equally spaced dots.
d) A series of twenty dashes.

3. How should the signal from an (AIS-SART’s) Search and Rescue Transmitter appear on AIS?
a) An eight identical position report messages .
b) A series of dashes.
c) A series of spirals all originating from the range and bearing of the SART.
d) A series of twenty dashes.

4. How is "radio silence" imposed?


a) By the On Scene Coordinator (OSC).
b) By the Coast Earth Station (CES) controlling the distress communications on that frequency.
c) By the nearest Public Correspondence Coast Station.
d) By the vessel first responding to the distress call.

5. How are normal working conditions restored on a narrow band direct printing (NBDP) frequency on
which radio silence had been imposed?
a) The RCC that imposed the radio silence must transmit an NBDP message stating "SILENCE
FINI".
b) The CES that imposed the radio silence must transmit a NBDP message stating "SILENCE
FINI".
c) The Public Correspondence Station (PCS) that imposed the radio silence must transmit a narrow
band direct printing message on the distress frequency stating "SILENCE FINI".
d) The High Seas Service (HSS) that imposed the radio silence must transmit a narrow band direct
printing message on the distress frequency stating "SILENCE FINI".

***Updated by: 2/M LINO A. ARRIESGADO


Date: OCT. 10, 2018
1
6. How are normal working conditions restored after radio silence has been imposed?
a) The Rescue Coordination Center (RCC) that imposed the radio silence must transmit a voice
message on the distress frequency stating "SEELONCE FEENEE".
b) The Coast Earth Station (CES) that imposed the radio silence must transmit a voice message on
the distress frequency stating "SILENCE FINI".
c) The Public Correspondence Station (PCS) that imposed the radio silence must transmit a voice
message on the distress frequency stating "SILENCE FINI".
d) None of the above.

7. Who may initiate a message indicating that distress traffic has finished?
a) The relay ship or shore station
b) The Rescue Coordination Center controlling a search and rescue operation
c) The vessel in distress
d) No message is required; when no distress traffic has been observed for a minimum of fifteen
minutes, it may be assumed that the radio silence imposed has been lifted.

8. In-charge of distress traffic during SAR operations when all participating ships are merchant marine
vessels….
a) On scene commander (OSC)
b) Rescue Coordination Center (RCC)
c) Coordinator Surface Search (CSS)
d) Local User Terminal (LUT)

9. You are the first vessel to arrive at the scene of a distress. Due to the volume of traffic on the radio, you
are an unable to communicate with the vessel in distress. What action should you take?
a) Broadcast “ Seelonce Distress “.
b) Broadcast “ Charlie Quebec-Mayday Quiet”.
c) Switch to Flag Hoists.
d) Key the microphone three times in quick succession.

10. An RCC or RSC may pass information to OSC or CSS in a standard format called?
a) SITREP
b) AUSREP
c) JASREP
d) MAREP

11. Several merchant ships arriving at the scene of a distress incident, one of them must assume the duties of
the Coordination Surface Search (CSS) which of the following statement is TRUE?
a) The CSS must be established by mutual agreement between the ship concerned.
b) CSS duties are always assumed by passenger vessels, or tankers in that order of precedence.
c) A tank vessel should never be assigned CSS duties unless only tank vessels are present.
d) The first vessel to arrive at a distress incident is designated as CSS.

***Updated by: 2/M LINO A. ARRIESGADO


Date: OCT. 10, 2018
2
12. Which of the following frequencies have NOT been designated for “On-scene” communications in the
Global Maritime Distress and Safety System?
a) VHF Ch-16.
b) MF radiotelephony on 2182 kHz.
c) SITOR (NBDP) on 2182.0 kHz.
d) None of these.

13. Which of the following steps should be taken, if possible, when the vessel must be abandoned because
of a distress situation?
a) Alert the U.S. Coast Guard by using the survival craft's portable INMARSAT unit.
b) Program the SART and EPIRB to transmit the vessel's location and situation.
c) Place the SART and EPIRB in the "on" position and secure them to the survival craft.
d) No additional steps are needed as the SART and EPIRB will both automatically float free and
operate properly.

14. What causes the SART to begin a transmission?


a) When activated manually, it begins radiating immediately.
b) It is either manually or water activated before radiating.
c) After being activated either manually or automatically the SART responds to radar interrogation.
d) It begins radiating only when keyed by the operator.

15. What stations may transmit on the frequencies in which distress traffic is taking place?
a) All stations for which silence was imposed by the Rescue Coordination Center or station
appointed to coordinate search and rescue activities.
b) All stations which are aware of a distress traffic.
c) Any station not itself in distress.
d) Any station in distress.

16. What stations may NOT transmit on the frequencies in which distress traffic is taking place?
e) Any stations for which silence was imposed by the RCC during SAR.
f) All stations which are aware of a distress traffic.
g) Any station not itself in distress.
h) Any station in distress.

17. Coastal warning type D in the EGC Safetynet service refers to:
a) Meteorological forecast
b) Search and Rescue Operation
c) Meteorological warning
d) NAVTEX message

***Updated by: 2/M LINO A. ARRIESGADO


Date: OCT. 10, 2018
3
18. What is the system being used when a vessel in the vicinity and the aircraft conducting the search and
rescue?
a) Search and Rescue Coordination System
b) General radio communication
c) Bridge-to-bridge communication
d) On-scene communication

19. MSI Delta (D) means:


a) Navigational Warnings
b) Meteorological Warnings
c) Ice Reports
d) Search and Rescue Information

20. MSI Echo (E) means:


a) Navigational warnings
b) Meteorological forecasts
c) Meteorological warnings
d) SATNAV system messages

21. MSI Juliet (J) means:


a) Navigational warnings
b) Meteorological forecasts
c) Meteorological warnings
d) SATNAV system messages

22. Which of the following is the primary frequency that is used exclusively for NAVTEX broadcasts
internationally?
a) 518 kHz
b) 2187.5 kHz
c) 4209.5 kHz
d) VHF channel 16 when the vessel is sailing in Sea Area A1, and 2187.5 kHz when in Sea Area
A2.

23. What is the transmitting range of most NAVTEX stations?


a) Typically 50-100 nautical miles (90-180 km) from shore.
b) Typically upwards of 1000 nautical miles (1800 km) during the daytime.
c) It is limited to line-of-sight or about 30 nautical miles (54 km).
d) Typically 200-400 nautical miles (360-720 km).

24. What is the primary equipment for receiving Maritime Safety Information (MSI)?
a) EPIRB
b) SART
c) NAVTEX
d) INM-B
***Updated by: 2/M LINO A. ARRIESGADO
Date: OCT. 10, 2018
4
25. What equipment can be used to receive Maritime Safety Information?
a) NAVTEX, EGC receiver or HF SITOR (NBDP).
b) EGC receive, Inmarsat B or F77 terminal.
c) HF SITOR (NBD), Inmarsat B or NAVTEX
d) All of these answers are correct.

26. The Navtex message which ensures that the system is functioning normally is type:
a) Z
b) L
c) J
d) F

27. Type D or SAR information messages in the international NAVTEX system are always numbered…
a) 00
b) 11
c) 33
d) 99

28. In the preamble of the NAVTEX message, the identity of the transmitting station is given by technical
code:
a) B4
b) B3
c) B2
d) B1

29. In the international NAVTEX system, meteorological forecasts are identified by the subject indicator:
a) L
b) E
c) I
d) Z

30. In programming the NAVTEX receiver, the types of message which are always received and cannot be
rejected are…
a) A, B, Z
b) A, B, D
c) A, B, E
d) A, B, F

31. A NAVTEX message has been received onboard with preamble ZCZC PB25. this message contains
a/an…
a) Meteorological warning
b) navigational warning
c) SAR-information
d) additional navigational warning
***Updated by: 2/M LINO A. ARRIESGADO
Date: OCT. 10, 2018
5
32. A NAVTEX message has the preamble ZCZC TD00. what type of message is this?
a) navigational warning
b) meteorological warning
c) SAR information
d) weather forecast

33. Coastal warning type A in the EGC safetyNET service refers to:
a) Navigational warnings
b) Meteorological forecasts
c) Meteorological warnings
d) SATNAV system messages

34. Coastal warning type B in the EGC safetyNET service refers to:
a) Navigational warnings
b) Meteorological forecasts
c) Meteorological warnings
d) SATNAV system messages

35. Coastal warning type C in the EGC safetyNET service refers to:
a) Navigational warnings
b) Meteorological forecasts
c) Meteorological warnings
d) ICE Reports

36. Coastal warning type J in the EGC safetyNET service refers to:
a) SATNAV system messages
b) LORAN system messages
c) OMEGA system messages
d) Pilot service messages

37. In EGC system, the reception of MSI is free of charge and is provided by the:
a) FleetNET service
b) InterNET service
c) SafetyNET service
d) MarineNET service

38. Coastal warning type H in the EGC SafetyNET service refers to:
a) LORAN system messages
b) DECCA system messages
c) Pilot service messages
d) OMEGA system messages

***Updated by: 2/M LINO A. ARRIESGADO


Date: OCT. 10, 2018
6
39. Coastal warning type F in the EGC SafetyNET service refers to:
a) Meteorological forecast
b) meteorological warnings
c) Pilot service messages
d) Search and rescue services

40. Which message subject matter can be programmed to be rejected or disabled by the operator of a
NAVTEX receiver?
a) Navigational warnings.
b) Meteorological warnings.
c) Pilot Service Messages.
d) All of these.

41. Coastal warning type Z in the EGC SafetyNET service refers to:
a) Additional navigational warnings
b) No message on hand
c) Ice reports
d) Meteorological forecasts

42. Which satellite system promulgates Maritime Safety Information?


a) AMVER.
b) Inmarsat-C SafetyNET.
c) NAVTEX.
d) Inmarsat-M SES.

43. What information is promulgated by the international SafetyNET?


a) MSI.
b) Traffic Lists.
c) Priority Messages.
d) MARAD.

44. In NAVTEX messages, a garbled character is printed as a/an…


a) Asterisk
b) question mark
c) letter “X”
d) period

45. Which of the following frequency bands would most likely provide reliable communications between
two stations that are 100 miles (160 km) apart.
a) The low frequency (LF) band.
b) The medium frequency (MF) band.
c) The high frequency (HF) band.
d) The very high frequency band.

***Updated by: 2/M LINO A. ARRIESGADO


Date: OCT. 10, 2018
7
46. What is the frequency range for medium frequency (MF)?
a) 30-300 kHz
b) 300-3,000 kHz
c) 1,000-10,000 kHz
d) 10-30 MHz

47. What is the frequency range for very high frequency (VHF)?
a) 3-30 MHz
b) 300-3,000 kHz
c) 30-300 MHz
d) 10-30 MHz

48. What is the frequency range for high frequency (HF)?


a) 3-30 MHz
b) 300-3,000 kHz
c) 30-300 MHz
d) 10-30 MHz

49. What is the frequency range for Ultra High Frequency?


a) 3-30 MHz
b) 300-3,000 MHz
c) 30-300 MHz
d) 10-30 MHz

50. What is the frequency range for Super High Frequency?


a) 30-300 GHz
b) 300-3,000 MHz
c) 30-300 MHz
d) 3-30 GHz

51. What is the primary frequency range for long distance skywave communications?
a) 3-30 MHz
b) 300-3,000 kHz
c) 30-300 MHz
d) 10-30 MHz

52. With respect to Distress communications the MAIN function of the On Scene Co-ordinator (OSC) is to:
a) recover and switch off the EPIRB
b) deploy a SART to assist in homing
c) fix the position of the casualty using an EPIRB
d) Maintain communication with all Search and Rescue (SAR) facilities

***Updated by: 2/M LINO A. ARRIESGADO


Date: OCT. 10, 2018
8
53. What indication is given to the personnel of survival craft of the approach of another vessel?
a) The SART will provide a visual or audible indication of interrogation by a 3-cm radar.
b) The Satellite EPIRB will emit an audible signal.
c) The VHF portable radio will emit an audible alarm signal on 156.525 MHz.
d) The VHF portable will provide a visual indication.

54. Which equipment on the bridge is used for locating a SART?


a) 3cm (9GHz radar)
b) 10cm (3GHz radar)
c) Radio direction finder
d) Radar direction finder

55. What is the most important factor which determine the range of a VHF station?
a) The output power
b) The height of antenna
c) The power supply
d) Both a and b

56. When activated, the SART responds only to radar signals with a wavelength of:
a) 3 cm
b) 5 cm
c) 10 cm
d) 12 cm

57. While approaching towards a distress site you hear the message PRU-DONCE over the radio
telephone. What will you do?
a) Use that frequency only restricted working communication
b) Shift your radio guard to the working frequency that will be
c) Advise the sender of your course, speed, position, and ETA at the distress site
d) Resume base course and speed because the distress is terminated

58. Which statement is true regarding distress communications under GMDSS?


a) Distress communications by NBDP should be in the ARQ mode when in communications with
the Coast Guard or other coast stations.
b) The Rescue Coordination Center (RCC) is responsible for controlling a search and rescue
operation and will also coordinate the distress traffic relating to the incident.
c) The Rescue Coordination Center may appoint another station to coordinate distress traffic
relating to the incident.
d) All of these.

***Updated by: 2/M LINO A. ARRIESGADO


Date: OCT. 10, 2018
9
59. The frequency band used as downlink frequency from the satellite to SES is the:
a) 1525.0 - 1545.0 MHZ (1.5GHZ )
b) 6425.0 - 6443.0 MHZ ( 6GHZ )
c) 3600.0 - 3623.0 MHZ ( 4 GHZ )
d) 1626.5 - 1646.5 MHZ (1.6 GHZ)

60. The frequency band used as uplink frequency from SES to the satellite is the:
a) 6425.0 - 6443.0 MHZ ( 6GHZ )
b) 3600.0 - 3623.0 MHZ ( 4 GHZ )
c) 1626.5 - 1646.5 MHZ (1.6 GHZ)
d) 1525.0 - 1545.0 MHZ (1.5GHZ )

61. The book which aims to provide guidance to those who, during emergencies at sea ,may require
assistance or may be able to render assistance is?
a) MERSAR manual
b) IAMSAR manual
c) ITU list of coast station
d) Admiralty list of radio signals

62. The phases of emergency at sea established for classifying incidents and determining the actions to be
taken are the following except:?
a) Critical phase
b) Uncertainty phase
c) Distress phase
d) Alert phase

63. Which of the following would most likely not prevent a SART's signal from being detected?
a) The SART was properly mounted in the lifeboat and rescue personnel were monitoring the 3-CM
radar.
b) The rescue personnel were monitoring the 3-CM radar and the SART was mounted improperly in
the lifeboat.
c) The SART was mounted improperly in the survival craft and rescue personnel were monitoring
the 10-CM radar.
d) The rescue personnel were monitoring the 10-CM radar and the SART was properly mounted in
the lifeboat.

64. Which is not a function of a satellite under COSPAS-SARSAT using satellite EPIRBs?
a) After the EPIRB’s position is calculated using the Doppler shift COSPAS-SARSAT satellites
provide follow-on SAR communications.
b) Relayed satellite message includes the EPIRB ID number which provides a reference for
retrieval of vessel information from the shore database.
c) Doppler shift of EPIRB signal is measured and the EPIRB’s position is calculated.
d) Information received from EPIRBs is time-tagged and transmitted to any Local User Terminal in
the satellite's view.
***Updated by: 2/M LINO A. ARRIESGADO
Date: OCT. 10, 2018
10
65. Which of the following would best be used for visual detection of a distressed vessel?
a) An EPIRB's strobe light.
b) A 9-GHz SART's beacon.
c) A 121.5-MHz EPIRB beacon.
d) A 406-MHz EPIRB beacon.

66. The IAMSAR procedure word that is used to separate portions of a message or one message from
another is?
a) Break
b) Negative
c) Affirmative
d) Roger

67. The IAMSAR procedure word that indicates, "I must pause for a few seconds, standby for further
transmissions" is?
a) Wait
b) Affirmative
c) Negative
d) Break

68. Which of the following satellite systems is of particular importance to search and rescue missions under
GMDSS?
a) COSPAS/SARSAT.
b) AMSAT.
c) NASA/Arienne.
d) COMSAT.

69. A DSC call for public correspondence may be repeated on the same or another DSC channel, if no
acknowledgement is received within 5 min. Further call attempts should be delayed at least ____ if
acknowledgement is still not received?
a) 5 minutes
b) 10 minutes
c) 15 minutes
d) 20 minutes

70. Which is the key part of the search and rescue system under GMDSS?
a) COSPAS/SARSAT satellites.
b) AMSAT satellites.
c) NASA satellites.
d) US Space Agency satellites.

***Updated by: 2/M LINO A. ARRIESGADO


Date: OCT. 10, 2018
11
71. You are making a ship-to-shore public correspondence call and the coast station does not reply to your
VHF DSC call after a second time, how long must you wait before calling again?
a) 3 minutes
b) 5 minutes
c) 10 minutes
d) 15 minutes

72. The SART operates correctly when interrogated by navigational radars with an antenna height of 15
meters at a distance of at least…
a) 5 nautical miles
b) 8 nautical miles
c) 10 nautical miles
d) 15 nautical miles

73. Maximum range within which SART could be interrogated by Navigational Radar at 9 GHZ frequency.
a) 5 miles
b) 10 miles
c) 15 miles
d) 20 miles

74. When activated the SART responds to Navigational Radar at 9 GHZ frequency at least at a distance of:
a) 5 miles
b) 10 miles
c) 15 miles
d) 20 miles

75. When making a DSC public correspondence call, how long would you wait before attempting to make a
second call with a station which does not answer?
a) 3 minutes
b) 5 minutes
c) 10 minutes
d) 15 minutes

76. A polarization of a radio wave is determined by a:


a) Position of the aerial
b) length of the aerial
c) width of the aerial
d) height of the aerial

77. A VHF transmission range is mainly determined by:


a) The height of the aerial
b) the right position of the squelch-adjustment
c) the weather condition
d) all of the options
***Updated by: 2/M LINO A. ARRIESGADO
Date: OCT. 10, 2018
12
78. What is the most common type of antenna for GMDSS MF-HF?
a) Horizontally polarized whip antenna.
b) Long wire antenna.
c) Vertically polarized whip antenna
d) None of the above.

79. Why is an antenna tuner required for MF-HF transmissions?


a) The length of the physically-fixed antenna must be electrically matched to the intended
frequency of operation.
b) The antenna tuner calculates the proper spectrum band for the operator to use.
c) To ensure transmissions are restricted to legal marine frequencies.
d) The antenna tuner indicates whether the ionosphere is ready to reflect a transmission properly

80. A vertical quarter wave antenna with a good ground connection will:
a) Radiate omni-directionally.
b) Radiate directionally due to being grounded.
c) Not function due to being grounded.
d) Only be used in Satellite communications.

81. What is the purpose or function of the antenna coupler?


a) All of these answers are functions of the antenna coupler.
b) Electrically matching the antenna system to the transmit frequency.
c) Changing the overall wavelength of the antenna system (in addition to the fixed length whip.
d) Enabling maximum transmitted on the band chosen by the operator.

82. What advantage does a vertical whip have over a long wire?
a) It radiates equally well in all directions.
b) It radiates directionally for better propagation.
c) It radiates a strong signal vertically.
d) It radiates more signal fore and aft.

83. Which of the following statements about a VHF vertical antenna is true?
a) The longer a VHF antenna the greater the signal gain.
b) The radiation pattern is a cardioid.
c) Maximum radiation is directly overhead.
d) The radiation pattern is a figure eight.

84. A VHF transmission range is mainly determined by:


a) the height of the aerial
b) the power output
c) the weather condition
d) both a) and b)

***Updated by: 2/M LINO A. ARRIESGADO


Date: OCT. 10, 2018
13
85. What statement is true regarding the additional equipment carriage requirement imposed for the survival
craft of vessels over 500 gross tons?
a) Additional carriage of two radio equipped lifeboats aft.
b) A second radar transponder is required.
c) Four additional portable VHF radios are required.
d) The ability to communicate in all modes with any shore station.

86. What is the joint International Satellite-Aided SAR system?


a) MARAD
b) IMO
c) COSPAS/SARSAT
d) INMARSAT

***Updated by: 2/M LINO A. ARRIESGADO


Date: OCT. 10, 2018
14
GOC FOR GMDSS REVIEWER
COMPETENCE 1 Transmit and Receive information using GMDSS subsystem and equipment and fulfilling the
functional requirements of GMDSS (GOC)
KUP2 – THE MEANS TO PREVENT THE TRANSMISSION OF FALSE DISTRESS
ALERTS AND THE PROCEDURES TO METIGATE THEIR EFFECTS OF SUCH ALETS.

1. What action should you take after sending a false distress alert on VHF?
a) Send a DSC cancellation message on Ch-70.
b) Make a voice announcement to cancel the alert on Ch-16.
c) Make a voice announcement to cancel the alert on Ch-13.
d) Make a voice announcement to cancel the alert on Ch-22A.

2. What action should you take after sending a false distress alert on MF?
a) Make a voice announcement to cancel the alert on 2187.5 kHz.
b) Make a voice announcement to cancel the alert on 2174.5 kHz.
c) Make a voice announcement to cancel the alert on 2182.0 kHz.
d) Send another DSC alert and follow on with voice on 2182.0 kHz.

3. When a false distress alert is detected during transmission on MFDSC YOU should?
a) Switch off transmitter immediately switch equipment on set equipment to channel 1.6 and make
broadcast to ALL. STATION cancelling the false alert.
b) Switch off transmitter immediately switch equipment on, set equipment to 2182 khs and make
broadcast to all station cancelling the false alert
c) Switch off transmitter immediately switch equipment on, tune equipment to all HF RT distress
frequency and make broadcast to all station cancelling the false alert .
d) Switch off transmitter immediately switch equipment on prepare cancellation message on
Inmarsat terminal and prepare cancellation message on Inmarsat terminal and transmit message
to nearest CES

4. What action should you take after sending a false distress alert on MF?
a) Send another DSC alert on 2187.5 kHz. and follow on with voice on 2187.5 kHz.
b) No action is necessary.
c) Send a DSC alert on all 7 DSC frequencies and follow on voice on 2174.5 kHz.
d) Make a voice announcement to cancel the alert on 2182.0 kHz.

5. What action should you take after sending a false distress alert on 8 MHz?
a) Make an "ALL SHIPS" call on all 5 H.F. telex channels canceling the alert.
b) Make an "ALL SHIPS" call on 8291.0 kHz canceling the alert.
c) Make a "MAYDAY" call on 8414.5 kHz canceling the alert.
d) Make an "URGENT" call on 8614.0 kHz canceling the alert.

***Updated by: 2/M LINO A. ARRIESGADO


Date: OCT. 10, 2018
15
6. What action should you take after sending a false distress alert on 12577.0 kHz?
a) No action is necessary.
b) Make an "ALL SHIPS" call on all 5 H.F. telex frequencies canceling the alert.
c) Make an "ALL SHIPS" call on the associated 12 MHz J3E frequency canceling the alert.
d) Send a message to the nearest RCC via Inmarsat canceling the alert.

7. What action should you take after sending a false distress alert on Inmarsat-C?
a) Press the "Distress Hot Keys” then press the "cancel" key.
b) Select "Transmit" menu and send a cancel message via the CES used for the distress alert.
c) Both of the above.
d) None of the above.

8. IMO resolution A814919) embodies guidelines for avoiding?


a) Avoiding interference
b) False distress alert
c) Avoiding usage of vhf when communicating with another station during navigation
d) Avoiding unnecessary search and rescue operations

9. Which of the following statements concerning testing and maintenance of SARTs is true?
a) An at-sea GMDSS maintainer is not able to test a SART as it is hermetically sealed.
b) Testing a SART should be performed only in controlled environment as a test signal may be
misinterpreted as a genuine distress situation.
c) A SART's battery must be replaced within ninety (90) days after the expiration date imprinted on
the unit.
d) All of the above.

10. What does the DSC control unit do if the GMDSS Radio Operator fails to insert updated information
when initiating a DSC distress alert?
a) It will abort the transmission and set off an audible alarm that must be manually reset.
b) It will initiate the DSC distress alert but, as no information will be transmitted, rescue personnel
will not be able to identify the vessel, its position, or its situation.
c) It will initiate the DSC distress alert, and default information will automatically be transmitted.
d) It will initiate the DSC distress alert, but any station receiving it will have to establish contact
with the distressed vessel to determine its identity, position, and situation.

11. When attempting to contact other vessels on Channel 16:


a) Limit calling to 30 seconds.
b) If no answer is received, wait 2 minutes before calling vessel again.
c) Channel 16 is used for emergency calls only.
d) Limit calling to 30 seconds and if no answer is received, wait 2 minutes before calling vessel
again.

***Updated by: 2/M LINO A. ARRIESGADO


Date: OCT. 10, 2018
16
12. Which of the following channels and modes should be used when initiating a distress alert transmission?
a) Ch-6 DSC.
b) Ch-6 Radiotelephony.
c) Ch-13 Radiotelephony and Ch-16 DSC.
d) Ch-70 DSC.

13. The best way to test the Inmarsat-C terminal is?


a) Send a message to a shore terminal and wait for confirmation
b) Send message to another ship terminal
c) Compose and send a brief message to your own Inmarsat-C terminal
d) If the send lights flashes proper operation has been confirmed

14. Testing of two-tone radiotelephone alarm signal should be made.


a) 2182 Khz into an artificial antenna
b) On 2182 Khz and must be heard clearly under normal conditions at a range of 150 nautical miles.
c) On any frequency other than 2182 Khz into an artificial antenna
d) This should never be used expect under emergency condition

15. Which action is the most appropriate action for a GMDSS radio Operator to take in a distress situation
where immediate help is needed, but the vessel is not sinking nor need to be abandoned?
a) Transmit distress call by HF/MF/VHF DSC or Inmarsat
b) Transmit distress call by activating the radiotelegraph automatic alarm signal
c) Notify the RCC (Rescue Coordination Center) through VHF FM on Channel 13
d) Switch off EPIRB and SART manually

16. A DSC Distress Alert is received. What action should be taken?


a) Silence the alarm
b) Review the incoming message information.
c) Acknowledge by DSC
d) Both a) and b)

17. A DSC Distress Alert is received. What action should be taken?


a) Advise the Master.
b) Transmit a DSC acknowledgement.
c) Call the nearest Coast Guard Station.
d) No action is necessary.

18. The action to be taken after sending a false Inmarsat-C Distress Alert is to:
a) Send a cancellation message, to all stations, on Ch16
b) Send a cancellation message, to all stations, on 2182kHz
c) Send a cancellation message, to the RCC, using the same LES
d) Switch off the equipment only after contacting a Coast Station/RCC

***Updated by: 2/M LINO A. ARRIESGADO


Date: OCT. 10, 2018
17
19. The action to be taken after sending a false Distress Alert on 8414.5kHz, indicating J3E, is to:
a) Reset the DSC controller and send a cancellation message, to all stations, on Ch16/2182 kHz
b) Reset the DSC controller and send a cancellation message, on the 8MHz voice distress frequency
c) Switch off the equipment only after contacting a Coast Station/RCC
d) Reset the DSC controller and send a cancellation message, to all stations, on 2187.5 kHz

20. The action to be taken after sending a false Distress Alert on VHF Ch70 is to:
a) Reset the DSC controller and send a cancellation message, to all stations, on Ch16
b) Reset the DSC controller and send a cancellation message, to all stations, on 2182kHz
c) Reset the DSC controller and send a cancellation message, to all stations, on 2187.5kHz
d) Switch off the equipment only after contacting a Coast Station/RCC

21. The action to be taken after sending a false Distress Alert on 2187.5kHz, indicating J3E, is to:
a) Reset the DSC controller and send a cancellation message, to all stations, on 2182kHz
b) Reset the DSC controller and send a cancellation message, to all stations, on Ch16
c) Switch off the equipment only after contacting a Coast Station/RCC
d) Reset the DSC controller and send a cancellation message, to all stations, on 2177kHz

22. The action to be taken after sending a false Distress Alert by EPIRB is to:
a) Send a cancellation message, to all stations, on 2182kHz
b) Contact the appropriate RCC and cancel the alert
c) Send a cancellation message, to all stations, on Ch16
d) Send a cancellation message, to the RCC, using the same LES

23. If a GMDSS Radio Operator initiates a DSC distress transmission but does not insert a message, what
happens?
a) The transmission is aborted and an alarm sounds to indicate this data must be provided by the
operator.
b) The transmission is not initiated and "ERROR" is indicated on the display readout.
c) The transmission will be made with "default" information provided automatically.
d) The receiving station will poll the DSC unit of the vessel in distress to download the necessary
information.

***Updated by: 2/M LINO A. ARRIESGADO


Date: OCT. 10, 2018
18
GOC FOR GMDSS REVIEWER
COMPETENCE 1 Transmit and Receive information using GMDSS subsystem and equipment
and fulfilling the functional requirements of GMDSS (GOC)
KUP3 – SHIP REPORTING SYSTEM

1. The “Y” line in AMVER is ____


a) Used for any English language amplifying commends or remarks the vessel may wish to send
AMVER regarding its current voyage
b) Used to request relay of the AMVER report to certain other ship reporting system
c) Information line
d) Estimated average speed

2. Which characters are sent by the ship station by when opening HF Telex, to indicate a desire to send
message to AMVER?
a) OPR+
b) AMV+
c) HELP+
d) MSG+

3. The “X” line in AMVER report ?


a) Used to request relay of the AMVER report to certain other ship reporting system
b) Currrent coastal radio station or satellite
c) Used for any English language amplifying commends or remarks the vessel may wish to send
AMVER regarding its current voyage
d) Vessel name and call sign

4. How should port of departure eg. Liverpool, UK (53-25N, 003-00W) be written when preparing an
AMVER report?
a) J/LIVERPOOL, UK/53-25N/003-00W//
b) L/LIVERPOOL, UK/53-25N/003-00W//
c) M/LIVERPOOL, UK/53-25N/003-00W//
d) G/LIVERPOOL, UK/5325N/00300W//

5. AUSREP is an acronym for:


a) Australian Radio Reporting System
b) Australian Ship Reporting System
c) Australian vessel reporting System
d) Australian Vessel rescue reporting System

***Updated by: 2/M LINO A. ARRIESGADO


Date: OCT. 10, 2018
19
6. AMVER is an acronym for:
a) American Vessel Assistance Rescue System
b) Automated Mutual-Assistance Vessel Rescue System
c) Automated Vessel Assistance Rescue System
d) Automatic Mutual-Assistance Rescue System

7. How should longitude 116 deg. 24.3W be written when preparing an AMVER report?
a) 116°24.3W
b) 11624W
c) 116.4W
d) 116W

8. When rendering position reports in the INMARSAT system, the two digit- code to use is:
a) 43
b) 42
c) 41
d) 51

9. What is the Automated Mutual-Assistance Vessel Rescue System?


a) A voluntary organization of mariners who maintain radio watch on 500khz 2182khz and
156.800mhz
b) An international system operated by the US Coast guard providing coordination of search and
rescue efforts.
c) A COORDINATED RADIO DIRECTION FINDING EFFORT BETWEEN THE Vederal
Communications Commission and US Coast guard to assist ships in distress.
d) A satellite based distress and safety alerting program operated by the U.S Guard

10. JASREP is an acronym for:


a) Japanese Ship Reporting System
b) Japanese Vessel Reporting Sysem
c) Japanese Radio Reporting System
d) Japan Ship Reporting System

11. How should date and time for 1200 (GMT) on the 20th be written when preparing JASREP report?
a) 1200UTC20
b) 20TH1200Z
c) 201200UTC
d) 201200Z

***Updated by: 2/M LINO A. ARRIESGADO


Date: OCT. 10, 2018
20
12. You ship is participating in the AMVER reporting system which of the following report your arrival at
your destination?
a) AMVER/DR//
b) AMVER/SR//
c) AMVER/PR//
d) AMVER/FR//

13. When a ship has been overdue at destination or it failed to make an expected or safety report?
a) AMVER/SP//
b) AMVER/PR//
c) AMVER/DR//
d) AMVER/FR//

14. Where Navtex cannot be feasibly established what system can be implemented to provide an automated
service in coastal waters to receive MSI?
a) Safety /Net
b) Amver
c) VHF/DSC
d) ARQ/SITOR

15. Which satellite system promulgates Maritime Safety information?


a) AMVER
b) INMARSAT –C Safety Net
c) Navtex
d) Inmarsat –M SES

16. An International safety service, which broadcasts Maritime Safety Information, such as Meteorological
and Hydrographic messages to all ships in certain geographical areas.
a) EGC Fleetnet
b) EGC SafetyNet
c) Navtex Service
d) EGC Service

17. An International commercial subscription service, allowing shipping companies and government bodies
to broadcasts messages to selected group of vessels.
a) EGC Fleetnet
b) EGC SafetyNet
c) Navtex Service
d) EGC Service

***Updated by: 2/M LINO A. ARRIESGADO


Date: OCT. 10, 2018
21
GOC FOR GMDSS REVIEWER
COMPETENCE 1 Transmit and Receive information using GMDSS subsystem and equipment
and fulfilling the functional requirements of GMDSS (GOC)
KUP4 – RADIO MEDICAL SERVICES

1. Concept of EMS(Emergency Medical Services) in emergency situation:


a) Coordinated radio medical communication between the ship and appropriate medical
authorities ashore
b) Sending urgency message to the coast station
c) Sending safety message to alert all ships
d) Sending distress call to the RCC

2. Concept of EMS(Emergency Medical Services) in emergency situation:


a) Sending Medical Advice
b) Sending urgency message to the coast station
c) Sending safety message to alert all ships
d) Sending distress call to the RCC

3. In Inmarsat operation what is the two- digit code which corresponds to medical advice :
a) 41
b) 42
c) 38
d) 32

4. Radio medical advice is available as :


a) Free of charge to ships of all nationalities
b) Chargeable per word
c) Minimum of 10 words as chargeable
d) All of the above

5. Which of the following maintenance functions is not the responsibility of the GMDSS Radio
Operator?
a) Visual inspection of equipment, including the antenna and associated components.
b) Perform on-the-air verification checks.
c) Perform scheduled testing of the battery's charged condition.
d) Aligning the power output stage for maximum power.

6. Which of the following service or maintenance functions may NOT be performed by the holder of a
GMDSS Radio Operator License?
a) Reset tripped circuit breakers or replace defective fuses.
b) Routine battery maintenance if used as part of the GMDSS station.
c) Any adjustments or maintenance that may affect the proper operation of the station.
d) Replacement of consumable items such as paper, ribbons, etc.
***Updated by: 2/M LINO A. ARRIESGADO
Date: OCT. 10, 2018
22
7. The following are the safety measure to be taken during handling storage battery in substation except
for?
a) Do not allow the battery to stand idle for long time.
b) Do not bring a flame inside the room.
c) Do not generate a spark inside the room.
d) Wear plash proof goggles, rubber gloves, while working with the battery.

8. A frequent maintenance is the basis for a reliable working condition of the battery, when working on
batteries, effective safety precautions must be taken except for;
a) Avoid over-discharging below 2.1 Volts for any cell
b) Ensure electrolyte level is maintained, but do not overfill, 1 cm above plates is adequate
c) Keep cells top clean and dry, check ventilation holes, tighten terminals and coat with
Vaseline
d) Charge the batteries for more than specified.

9. A treatment aimed at preventing the death or further damage to health of an ill or injured person
perceived to be in a life-threatening condition:
a) Hospital confinement
b) General check-up
c) First aid
d) Outpatient confinement

10. What are the Radiofrequency radiation that generate energy via water molecules results in rapid
heating through an object?
a) Microwave radiation
b) Radioactive radiation
c) X-radiation and gamma radiation
d) Thermal radiation

11. In inmarsat operation what is the two-digit code which corresponds to medical assistance.
a) 42
b) 32
c) 38
d) 41

12. Two important data/factors needed before a DOCTOR will visit a ship at sea to further examine the
patient/crew aboard:
a) Sailing Plan
b) Position report
c) E.T.A.
d) All of the above

***Updated by: 2/M LINO A. ARRIESGADO


Date: OCT. 10, 2018
23
13. At the first onset of illness of the crew at sea, the medical services, as provided by law is mandatory,
regardless, of ship’s nationality, the master of the ship must immediately send a Radio Medical
Advice to the shore authorities, for the purpose of:
a) Complying the regulation
b) Inhibit himself from any complaint
c) To save life of the crew
d) None of the above

14. What is the most important component of a Radio Medical advice message which serves as a
consultation guide for the Physician to visualize the patient’s actual health condition:
a) Radio medical consultation form (P18)
b) Radio medical advice
c) Radio medical assistance
d) Radio medical Insurance form

15. If a seafarer is inflicted with serious illness onboard and need immediate hospitalization ashore what
category of urgency message the Master will send ashore :
a) Distress message
b) Urgency message
c) Routine message
d) Medical assistance/medical evacuation

16. In an instance, where a seafarer is contracted with a dreaded disease onboard, where ,a Second
Officer is acting as a doctor, and he does not know what specific medicine to be given, what
category of urgency message the Master will send ashore :
a) Email message
b) Medical assistance
c) Medical advice message
d) Urgency message

17. In the event that a life of a seafarer is in imminent danger and in the brink of death, due to serious
illness, which, priority of message the master will transmit:
a) Distress message
b) Urgency message
c) Safety message
d) Radio medical advice

18. The vessel, most likely, is carrying what type of medical Kit :
a) Type A
b) Type B
c) Type C
d) Type D

***Updated by: 2/M LINO A. ARRIESGADO


Date: OCT. 10, 2018
24
19. In an emergency, the vessel carrying a “Patient” onboard, without delay, must communicate
immediately to what specific/appropriate authorities ashore:
a) RCC
b) Coast Earth Station
c) COAST STATION
d) RCC with affiliated hospital as mandated by administration

20. On the first communication of the vessel seeking a medical advice what document is primarily
needed by the hospital before giving final treatment to the patient:
a) Radio Medical Consultation Form
b) Radio Medical Advice
c) Radio Pratique
d) Emergency Medical Message

21. What is a “Radio Medical Consultation” means :


a) Admitting patients in hospital
b) Admitting patients from any sector
c) Outpatients from hospital
d) 24 hours –a- day- free radio communication service for all ships seeking medical advice
while at sea.

22. In GMDSS Radio medical services is designed for the following incidents:
a) Seafarers who are sick onboard ship
b) Persons who are sick ashore
c) Persons who are sick in their home
d) Patients who are sick in the hospital

23. Medical advice by radio may be obtained in one of two ways :


a) From a shore station, link to a hospital.
b) From another ship carrying a doctor
c) Both A and B
d) None of the above

24. Radio Medical Support System (RMSS) is available to all ships at sea for:
a) 24 hrs
b) 16 hrs
c) 12 hrs
d) 8 hrs

***Updated by: 2/M LINO A. ARRIESGADO


Date: OCT. 10, 2018
25
25. Transmission of medical advice, before it is transmitted, is approved by :
a) Medical doctor on board
b) S.S.O.
c) Master
d) Second officer

26. In case, the Radio Officer onboard has no complete reference/addresses of the accredited hospitals
ashore authorized for admission of crew-patient from the ship, he must send the radio medical advice
message to the:
a) Any RCC in the area
b) COAST STATION in DSC
c) ANY COAST EARTH STATION WITH AFFILLIATED HOSPITAL
d) Network coordination station

27. After the vessel has reported/transmitted the complete Radio Medical Communication containing the
crew/patient’s general symptom, the hospital ashore will dispatch, what expertise/specialization of a
doctor to the ship:
a) Doctor specialized in a certain field
b) Doctor specialized according to patient’s illness
c) General internal medicine physician
d) Pediatric expertise physician

28. E.M.S. begins with:


a) Emergency medical incident detection onboard
b) Emergency situation at sea
c) Sending medical evacuation to the shore
d) All of the above

29. What institution is responsible in establishing the EMS


a) INMARSAT
b) COSPAS-SARSAT
c) IMO
d) Administration

30. For the purpose of INFORMATION SECURITY of all personal data taken or gathered from all
patients onboard ship worldwide medical authorities develop a DATABASE to keep all records in
stringent confidentiality into a computer system. The program is called:
a) RMSS
b) SEMPOS
c) GMDSS
d) SOLAS

***Updated by: 2/M LINO A. ARRIESGADO


Date: OCT. 10, 2018
26
31. To protect unauthorized access to the RMSS where sensitive information of the patient is stored, the
law mandated that all doctors who have an access the system must have:
a) Doctor must be registered with the RMSS SYSTEM
b) Issued a unique key or password
c) Certified by the computer
d) All of the above

32. Statistically, most medicines used onboard the vessel contained properties to cure certain illnesses,
such as:
a) Cholesterol
b) High blood
c) Hypertension
d) Analgesic and antibiotic properties

33. Radio Medical Consultation Services is provided by law based on a recommendation adopted at the
general conference of the International Labor Organization (ILO) in the year:
a) 1992
b) 1966
c) 1958
d) 1912

34. Sending medical advice using INMARSAT, while the vessel is in Japanese territorial waters what is
the recipient affiliated hospital providing the free services:
a) Tokyo General Hospital
b) Kobe General Hospital
c) Osaka General Hospital
d) Yokohama Seamen’s Insurance Hospital

35. The correct DSC Call category to be used prior to requesting a Medical Evacuation is:
a) Pan Pan
b) Mayday
c) Distress
d) Urgency

36. To understand clearly the flow of traffic in sending radio medical advice to the shore, is by way of
transmitting to:?
a) LUT TO MRCC
b) LES T RCC
c) LES direct to affiliated hospital
d) LES to subscriber

***Updated by: 2/M LINO A. ARRIESGADO


Date: OCT. 10, 2018
27
37. In GMDSS, Radio Medical Services is stipulated under column-2 of table A-IV/2 of STCW code is
listed under what sequence no:?
a) .1
b) .2
c) .3
d) .4

38. In sending Radio Medical advice how many essential medical data are required by the Radio
Medical Center hospital to diagnose the patient’s health condition:
a) 5
b) 8
c) 9
d) 10

39. In the EMS system, how many “key-points” communications are required for essential and effective
EMS communication:
a) 3
b) 10
c) 7
d) 5

40. Medical kits carried by ships are classified into how many types:
a) 3
b) 4
c) 5
d) 6

41. When the DOCTOR is onboard ship how many questions he would probably ask the patient to
diagnose his illness:
a) 10
b) 5
c) 7
d) 8

42. As far as the “symptoms-pain” is concern the Doctor will go back again and ask how many questions
to the crew-patient:
a) 10
b) 6
c) 8
d) 5

***Updated by: 2/M LINO A. ARRIESGADO


Date: OCT. 10, 2018
28
43. Doctor’s visit to the patient at sea is arranged by:
a) Master
b) Charterer
c) Agent
d) Owner

44. What year seamen’s insurance law started:


a) Feb 1, 1992
b) Feb 1, 1999
c) April 1, 1963
d) Feb 1, 1995

45. What are the type of electromagnetic radiation with enough energy to ionized biological matter.?
a) X-radiation and gamma radiation
b) Chemical reactive radiation
c) Radioactive radiation
d) Thermal radiation

46. What type of radiation are VHF and UHF radio signals?
a) Gamma radiation
b) Ionizing radiation
c) Alpha radiation
d) Non-ionizing radiation

***Updated by: 2/M LINO A. ARRIESGADO


Date: OCT. 10, 2018
29
GOC FOR GMDSS REVIEWER
COMPETENCE 1 Transmit and Receive information using GMDSS subsystem and equipment
and fulfilling the functional requirements of GMDSS (GOC)
KUP5 – USE OF INTERNATIONAL CODE OF SIGNALS AND THE IMO STANDARD
MARINE COMMUNICATION PHRASES

1. When the GMDSS Radio Operator on watch hears "SECURITE" spoken three times, he can expect to
receive the following information:
a) Message containing information concerning navigational warnings.
b) Safety of vessel or person is in jeopardy.
c) Vessel in need of immediate assistance.
d) Coast Station Traffic list.

2. How should the number "2" be pronounced when spoken on the radiotelephone?
a) NUM-BER-TOO
b) BEES-SOH-TOO
c) DOS-SOH-TU
d) BEE-SOH-TOO

3. How should the letter "D" be pronounced when spoken on the radiotelephone?
a) DUKE
b) DA VID
c) DOG
d) DELL TAH

4. The Radiotelephone Safety signal is:


a) "Securite" repeated 3 times.
b) "Safety Safety Safety".
c) "Pan Pan" repeated 3 times.
d) "Securite Securite" repeated 3 times.

5. How should the letter "I" be pronounced when spoken on the radiotelephone?
a) IN DEE GO
b) IN DEE AH
c) I EE
d) I VAN HO

6. If the Watch Officer hears "PAN PAN" spoken 3 times it means:


a) A navigation or important meteorological warning should follow.
b) The station is preparing to transmit an URGENT message possibly concerning the safety of a
mobile unit or person.
c) A Mobile unit is in need of immediate assistance.
d) None of the above.
***Updated by: 2/M LINO A. ARRIESGADO
Date: OCT. 10, 2018
30
7. The Radiotelephone Safety signal is:
a) "Securite" repeated 3 times.
b) "Safety Safety Safety".
c) "Pan Pan" repeated 3 times.
d) "Securite Securite" repeated 3 times.

8. How should the number "7" be pronounced when spoken on the radiotelephone?
a) SAY-TAY-SEVEN
b) SEE-ETA-SEVEN
c) NUM-BER-SEVEN
d) NEW-MER-AL-SEVEN

9. The radiotelephone distress message consist of :


a) MAYDAY spoken three times, call sign and name of vessel in distress.
b) Particulars of its position, latitude and longitude, and other information which might facilitate
rescue, such as length, color and type of vessel, number of persons on board.
c) Nature of distress and kind of assistance desired.
d) All of these.

10. Which of the following situations would normally use the Safety priority?
a) Loss of 5 containers with lashing gear over the side.
b) Treatment of crewmember breaking a leg in a cargo hold.
c) A fire in the generator flat/spaces.
d) Answers a) and b) are both possible.

11. If you are in urgent need of helicopter, which signal code could you send?
a) BR
b) BS
c) BT1
d) BZ

12. How should the letter "R" be pronounced when spoken on the radio telephone?
a) ROW ME OH
b) B.AR AH
c) C.ROA MA O
d) D.AR EE

13. How should the letter "V" be pronounced when spoken on the radiotelephone?
a) VIK TAH
b) VIC TO RE
c) VIX TOO RE
d) VEE

***Updated by: 2/M LINO A. ARRIESGADO


Date: OCT. 10, 2018
31
14. How should the number "1" be pronounced when spoken on the radiotelephone?
a) OO-NO
b) OO-NAH-WUN
c) NUM-EV-WUN
d) NEW-MAL-WON

15. How should the number "3" be pronounced when spoken on the radiotelephone
a) TAY-RAH-TREE
b) BEES-SOH-THREE
c) NUM-ERL-THREE
d) TRIC-THREE

16. The single-letter signal for man overboard is:


a) “ X ”
b) “ V ”
c) “ O “
d) “ B “

17. What is the single letter signal used when you wish to communicate with another vessel?
a) K
b) Y
c) L
d) Z

18. Which of the Two-Letter signals are the most commonly used as “I require immediate assistance”?
a) AL
b) AN
c) CD
d) FR

19. In the use of visual signaling, which Morse Code regular procedural signal are to be applied using two letters
code for waiting signal?
a) AA
b) AB
c) AR
d) AS

20. The two-letter signal which means “I am on fire” is:


a) AC
b) YU
c) IT
d) CD

***Updated by: 2/M LINO A. ARRIESGADO


Date: OCT. 10, 2018
32
21. Which of the Two-Letter signals are the most commonly used as “I require immediate assistance”?
a) CD
b) AL
c) AN
d) FR

22. In ICS, the signal for the phrase “I am sinking” is:


a) DS
b) PX
c) DX
d) TX

23. In facilitating visual signaling, what are the most commonly used important Two-letter signals which
specify “I have a doctor onboard”?
a) AL
b) AN
c) CD
d) FR

24. In facilitating visual signaling, what are the most commonly used important Two-letter signals which
specify “I need a doctor”?
a) AL
b) AN
c) CD
d) FR

25. In facilitating visual signaling, what are the most commonly used important Two-letter signals which
specify “I am abandoning my vessel”?
a) AC
b) DV
c) JL
d) NC

26. In facilitating visual signaling, what are the most commonly used important Two-letter signals which
specify “I am drifting”?
a) AC
b) DV
c) JL
d) NC

***Updated by: 2/M LINO A. ARRIESGADO


Date: OCT. 10, 2018
33
27. In facilitating visual signaling, what are the most commonly used important Two-letter signals which
specify “Man overboard, take action to pick him up”?
a) GW
b) PP
c) EF
d) QX

28. In facilitating visual signaling, what are the most commonly used important Two-letter signals which
specify “SOS/MAYDAY has been cancelled”?
a) NC
b) EF
c) QQ
d) RU

29. In facilitating visual signaling, what are the most commonly used important Two-letter signals which
specify “Keep well clear of me”?
a) GW
b) PP
c) EF
d) QX

30. In facilitating visual signaling, what are the most commonly used important Two-letter signals which
specify “Keep clear of me, I am manoeuvering with difficulty”?
a) RU
b) QQ
c) SO
d) QX

31. In facilitating visual signaling, what are the most commonly used important Two-letter signals which
specify “I require health clearance”?
a) RU
b) QQ
c) SO
d) QX

32. In facilitating visual signaling, what are the most commonly used important Two-letter signals which
specify “Your navigation lights is not visible”?
a) PD
b) PP
c) QD
d) QT

***Updated by: 2/M LINO A. ARRIESGADO


Date: OCT. 10, 2018
34
33. In facilitating visual signaling, what are the most commonly used important Two-letter signals which
specify “I am going ahead”?
a) PD
b) PP
c) QD
d) QT

34. In facilitating visual signaling, what are the most commonly used important Two-letter signals which
specify “I am going astern”?
a) PD
b) PP
c) QD
d) QT

35. In facilitating visual signaling, what are the most commonly used important Two-letter signals which
specify “I request permission to anchor”?
a) QU
b) QX
c) SO
d) UM

36. In facilitating visual signaling, what are the most commonly used important Two-letter signals which
specify “ Anchoring is prohibited”?
a) QU
b) QX
c) SO
d) UM

37. In facilitating visual signaling, what are the most commonly used important Two-letter signals which
specify “ You should stop your vessel instantly ”?
a) SO
b) UM
c) UP
d) YU

38. In facilitating visual signaling, what are the most commonly used important Two-letter signals which
specify “ Permission to enter Harbour is urgently requested. I have an emergency”?
a) SO
b) UM
c) UP
d) YU

***Updated by: 2/M LINO A. ARRIESGADO


Date: OCT. 10, 2018
35
39. In facilitating visual signaling, what are the most commonly used important Two-letter signals which
specify “ The Harbour is closed to traffic”?
a) SO
b) UM
c) UP
d) YU

40. In facilitating visual signaling, what are the most commonly used important Two-letter signals which
specify “ I am going to communicate with your station by means of the International Code of Signals”?
a) SO
b) UM
c) UP
d) YU

41. As a secondary method of alerting, what names are used to prevent the reception of unwanted broadcast
s by vessel’s utilizing the NAVTEX system?
a) Select all stations
b) Select all types of messages
c) Contact the NAVTEX coordinator
d) Programming the receiver to reject unwanted broadcasts

42. To exhibit a distress signal without using the GMDSS sub-system (equipment), which of the following
should be executed as to get attention of nearby vessels?
a) Parachute red flare
b) The use of EPIRB
c) The use of Navtex receiver
d) None of the above

43. Under GMDSS, a compulsory VHF-DSC radiotelephone installation must be tested at what minimum
intervals at sea?
a) Annually, by a representative of the FCC.
b) Daily
c) At the annual SOLAS inspection.
d) Monthly

44. What is the requirement for emergency and reserve power in GMDSS radio installations?
a) Compulsory ships must have emergency and reserve power sources for radio communications.
b) An emergency power source for radio communications is not required if a vessel has proper
reserve power (batteries).
c) A reserve power source is not required for radio communications.
d) Only one of the above is required if a vessel is equipped with a second 406 EPIRB as a backup
means of sending a Distress alert.

***Updated by: 2/M LINO A. ARRIESGADO


Date: OCT. 10, 2018
36
45. What is the proper procedure to be followed upon receipt of a distress alert transmitted by use of Digital
Selective Calling Techniques?
a) Set watch on the DSC alerting frequency in the band of frequencies the alert was received.
b) Set a continuous watch on VHF-FM Channel 13, 16 and DSC on Channel 70.
c) Ship stations equipped with narrow-band direct-printing equipment should respond to the
Distress alert as soon as practicable by this means.
d) Set watch on the radiotelephone Distress and Safety frequency associated with the Distress and
Safety calling frequency on which the Distress alert was received.

46. Transmission of a Distress alert by a station on behalf of another vessel actually in Distress should not
occur:?
a) When communications between the Distress vessel and a Coast station are already in progress.
b) When the mobile unit actually in Distress is not itself in a position to transmit the Distress alert.
c) When the Master or responsible person on the mobile unit not in Distress so decides.
d) When the responsible person at the Coast Station determines further help is necessary.

47. In visual signaling, what method of communication uses two hand flags to indicate letters of the
alphabet?
a) Flag signaling
b) Flashing light signaling
c) Semaphore or hand flags or arms signaling
d) Sound signaling

48. A ship at anchor has been communicating effectively with a shore station approximately 500 miles (805
km) distant on a frequency in the 16 MHz band periodically throughout the day. Toward the late
afternoon and evening, what effect should be noticed?
a) Communications should be maintained with slight improvement in the signal received from the
shore station.
b) The grey line effect will prevent communication after dark.
c) Communications should gradually deteriorate and become impossibly on this frequency at night.
d) Communications should improve and peak at night.

49. One of the signals, other than a distress signal, that can be used by a rescue boat to attract attention is
a/an__________ .?
a) burning barrel
b) Search light
c) red star shell
d) orange smoke signal

***Updated by: 2/M LINO A. ARRIESGADO


Date: OCT. 10, 2018
37
GOC FOR GMDSS REVIEWER
COMPETENCE 1 Transmit and Receive information using GMDSS subsystem and equipment
and fulfilling the functional requirements of GMDSS (GOC)
KUP6 –THE ENGLISH LANGUAGE, BOTH WRITTEN AND SPOKEN, FOR THE
COMMUNICATION OF INFORMATION RELEVANT TO SAFETY OF LIFE AT SEA

1. Being specific is the opposite meaning of ____.


a) Overall
b) General
c) Common
d) Group

2. One of these is not a Radio Equipment.


a) Skillet
b) Transmitter
c) Receiver
d) Radio

3.Our ship sailed the world ____the best crew from the Philippines.
a) Come
b) With
c) of
d) include

4.The word FORE is the opposite of ____.


a) Port
b) Starboard
c) Upward
d) AFT

5.The bridge deck surface was so _____that it took us more than a day to clean.
a) Dusty
b) Shinny
c) Rusty
d) Dirty

6. Hands signal also means…


a) USE FLAG TO COMMUNICATE
b) USE HAND TO COMMUNICATE
c) USE SIGNAL TO COMMUNICATE
d) USE NUMBER SIGN TO COMMUNICATE

***Updated by: 2/M LINO A. ARRIESGADO


Date: OCT. 10, 2018
38
7. _____will you do if you see a man overboard?
a) WHEN
b) WHERE
c) WHO
d) WHAT

8. The_____is the safest way or link going to the ship’s accommodation?


a) ACCOMMODATION
b) GANGWAY
c) LADDER
d) STAIRWAY

9. Chief Officer would not allow you to enter ____a permit.


a) IDENTITY
b) WITHOUT
c) ACCEPTANCE
d) APPROVAL

10.When you see a vessel raising a flag J what will you do?
a) Keep clear because the ship is big
b) Keep clear because the ship is on fire
c) Keep clear because the ship is aground
d) Keep clear because the ship is not in command

11.What is the meaning of letter K in radio communication?


a) I wish to communicate with you
b) I like to communicate with you
c) I don’t like to communicate to you
d) I want to talk to you by radio

12.OVER means the work is done.


a) SEPARATE
b) CONTINUING
c) RESISTIVE
d) FALSE

13. Working on the Radar antenna means ____.


a) Working the radar aloft
b) Not fixing the antenna
c) Working the radio
d) Working the mast

***Updated by: 2/M LINO A. ARRIESGADO


Date: OCT. 10, 2018
39
14. The warning note of Radar equipment says, “ man is working aloft” means?
a) Someone is working on the radar mast
b) Working on the antenna
c) Working on the satellite
d) Working on the radio

15. Altering course to Starboard?


a) DELTA
b) INDIA
c) ECHO
d) FOXTROT

16. Altering course to Port?


a) LIMA
b) INDIA
c) KILO
d) ALFA

17. Make a voice communication on VHF____not on Ch. 13 or Ch. 06 in radiotelephony.


a) CH. 12
b) CH. 13
c) CH. 08
d) CH. 16

18. What time they will ___ discharge tomorrow?


a) START
b) BEGAN
c) RESTART
d) COMMENCE

19. 2356.3 Hrs.


a) REAL DAYS
b) REAL TIMES
c) REAL MONHTS
d) REAL YEARS

20. Unique:
a) SPECIAL
b) RARE
c) UNCOMMON
d) NOT C0MM0N

***Updated by: 2/M LINO A. ARRIESGADO


Date: OCT. 10, 2018
40
21. . Normal:
a) ORDINARY
b) COMMON
c) GENERAL
d) POPULAR

22. Future:
a) COMING
b) NEXT
c) AFTER
d) WILL

23. Break
a) WAIT
b) PAUSE
c) STANDBY
d) WITHOLD

24. Opposite of sincere


a) Artificial
b) Feigned
c) insincere
d) All of the above

25. ______to be used to announce a distress message


a) DISTRESS
b) PAN PAN
c) SECURITE
d) MAYDAY

26. ______to be used to announce an urgency message


a) MAYDAY
b) SECURITY
c) URGENCY
d) PAN PAN

27. ______to be used to announce a safety message


a) SAFETY
b) PAN PAN
c) SECURITE
d) MAYDAY

***Updated by: 2/M LINO A. ARRIESGADO


Date: OCT. 10, 2018
41
28. Opposite of exactly :
a) Inaccurate
b) Indefinite
c) wrong and doubtful
d) Any of the above

29. Meaning of Exactly:


a) All answers are correct
b) Accurate
c) definitely
d) correct

30. Reconstruct the sentence correctly? commenced ship at 1400H loading….


a) Commence loading ship at 1400H
b) At 1400H ship commence loading
c) The ship loading commenced at 1400H
d) None is correct

31. Which radiotelex command would you use to retrieve the message stored at the coastal station.?
a) SVC+
b) TGM+
c) MSG+
d) DIRTLXOXY+

32. You have a Manoverboard situation, what would be the priority of the DSC call you are going to send?
a) Safety
b) Distress
c) Urgency
d) Routine

33. For a DSC Distress call the distress information is contained in four messages, Message 1 is?
a) Distress coordinates message
b) Time Indication (UTC)
c) Nature of distress message
d) Single character to indicate the type of communication which is prepared by the station in
distress for subsequent communication.

34. Message 2 in the distress call contain the.?


a) Time indication (utc) when the coordinates were valid
b) Nature of Distress message
c) Distress coordinates message
d) Single character to indicates the type of communication which is prepared by the station in
distress for subsequent communication.

***Updated by: 2/M LINO A. ARRIESGADO


Date: OCT. 10, 2018
42
35. Message 3 in the DSC distress call contains the:
a) Time indication (UTC) when the coordinates were valid
b) nature of distress message
c) distress coordinates message
d) single character to indicate the type of communication which is preferred by the station in
distress for subsequent exchange of distress traffic

36. Message 4 in the DSC distress call contains the:


a) time indication (UTC) when the coordinates were valid
b) nature of distress message
c) distress coordinates message
d) Single character to indicate the type of communication which is preferred by the station in
distress for subsequent exchange of distress traffic

37. What element of a DSC call indicates the type of distress situation?
a) Message 1 of distress calls
b) Message 2 of distress calls
c) Message 3 of distress calls
d) Message 4 of distress calls

38. What element of a DSC call describes the emergency position with the aid of 10 digits?
a) Message 1 of distress calls
b) Message 2 of distress calls
c) Message 3 of distress calls
d) Message 4 of distress calls

39. What element of a DSC call describes the hour of the position, and given in UTC with the aid of 4
digits?
a) Message 1 of distress calls
b) Message 2 of distress calls
c) Message 3 of distress calls
d) Message 4 of distress calls

40. What element of a DSC call indicates the type of communication desired the subsequent
correspondence?
a) Message 1 of distress calls
b) Message 2 of distress calls
c) Message 3 of distress calls
d) Message 4 of distress calls

***Updated by: 2/M LINO A. ARRIESGADO


Date: OCT. 10, 2018
43
41. What element of the DSC call indicates whether one wants the communication by telephony, telex or
data transfer?
a) Message 1 of routine calls
b) Message 2 of routine calls
c) Message 3 of routine calls
d) Message 4 of routine calls

42. What element of the DSC call indicates the frequency desired for the subsequent communication?
a) Message 1 of routine calls
b) Message 2 of routine calls
c) Message 3 of routine calls
d) Message 4 of routine calls

43. What element of the DSC call which is transmitted as a control bit for the whole call?
a) End of sequence
b) Error check character
c) Phasing sequence
d) Format specifier

44. What element of the DSC call indicates if the call requires an acknowledgement of receipt and if the
current messages is a reply to another call?
a) End of sequence
b) Error check character
c) Phasing sequence
d) Format specifier

45. What element of the DSC call consists of dots sent out to enable the scanning receiver to tune itself to
the frequency (stop scanning)?
a) Dot pattern
b) Phasing sequence
c) End of sequence
d) Error check character

46. What element of the DSC call serves as the phasing-in signals preparing the receiver to receive the
information from the DSC transmitter?
a) Dot pattern
b) Phasing sequence
c) End of sequence
d) Error check character

***Updated by: 2/M LINO A. ARRIESGADO


Date: OCT. 10, 2018
44
47. The standard phrase which means “I have completed my transmission and am ready to receive yours”
is:
a) Over
b) Nothing more
c) Stay on
d) Out

48. The procedure word OUT indicates?


a) end of transmission when an immediate reply is expected
b) your transmission received satisfactorily
c) wait a few seconds, standby for further transmissions
d) End of transmission when no reply is expected or required

49. The IAMSAR procedure word that indicates, "I have received your transmission satisfactorily" is?
a) NEGATIVE
b) ROGER
c) AFFIRMATIVE
d) WAIT

50. The IAMSAR procedure word WAIT indicates:


a) end of transmission when no reply is expected or required
b) your transmission received satisfactorily
c) Pause for a few seconds, standby for further transmissions
d) end of transmission when an immediate reply is expected

51. The IAMSAR procedure word that indicates, "I must pause for a few seconds, standby for further
transmissions" is:
a) WAIT
b) NEGATIVE
c) AFFIRMATIVE
d) BREAK

52. The procedure word OVER indicates:


a) that the transmission was received satisfactorily
b) The end of transmission when an immediate reply is expected
c) a short pause - standby for further transmissions
d) the end of transmission when no reply is expected or required

***Updated by: 2/M LINO A. ARRIESGADO


Date: OCT. 10, 2018
45
53. Which of the following region lies outside Sea Area A1, A2 and A3?
a) Pacific Ocean Region
b) Indian Ocean Region
c) Atlantic Ocean Region
d) Polar Region

54. Which of the following region lies outside Sea Areas A1, A2, and A3?
a) Sea Areas only apply to Inmarsat footprint areas.
b) Sea Area A3-I (Inmarsat coverage) and Sea Area A3-S (HF SITOR coverage).
c) There are no additional Sea Areas.
d) Sea Area A4

55. The COSPAS/SARSAT system makes use of low altitude satellites follow a/an:
a) Circular Orbit
b) Polar Orbit
c) Geostationary Orbit
d) Elliptical Orbit

56. What system provides accurate vessel position information to the GMDSS equipment?
a) GPS.
b) Cospas-Sarsat.
c) EPIRB.
d) Inmarsat-A

57. What is the primary equipment for receiving MSI.


a) SART.
b) EPIRB.
c) Navtex.
d) Inmarsat-A

58. In GMDSS, VHF Channel 70 EPIRB can be a substitute for a Float-Free EPIRB in:
a) Sea Area A4
b) Sea Area A3
c) Sea Area A2
d) Sea Area A1

59. What sea area is defined as being within range of a shore-based MF station that provides for continuous
DSC alerting?
a) Sea area A2.
b) Coastal waters.
c) Sea area A3.
d) Sea area A1.

***Updated by: 2/M LINO A. ARRIESGADO


Date: OCT. 10, 2018
46
60. What sea area is defined as within the range of shore-based MF stations which provides continues DSC
alerting on 2187.5 Khz distress and safety calling frequency?
a) Sea Area A1
b) Sea Area A2
c) Sea Area A3
d) Sea Area A4

61. If a vessel is engaged in local trade and at no point in its voyage travels outside of the range of a VHF
shore station with continuous DSC alerting then the vessel is operating in what area?
a) Coastal and international zones.
b) Inland and coastal waters.
c) Sea areas A1 and A2.
d) Sea area A1.

62. What is defined as an area, excluding sea areas A1 and A2, within the coverage of an Inmarsat
geostationary satellite in which continuous alerting is available?
a) Ocean Area Regions AOR-E, AOR-W, POR or IOR.
b) Sea Area A4.
c) Sea Area A3.
d) Coastal and Inland Waters.

63. SITOR equipment is a full, partial or alternate carriage requirement under GMDSS for vessels operating
in which sea area(s)?
a) A1.
b) A1 and A2.
c) A3 and A4.
d) A1, A2, A3 and A4.

64. What is defined as the area within the radiotelephone coverage area of at least one VHF coast station in
which continuous DSC alerting is available as defined by the IMO regulation for GMDSS?
a) Sea Area A1.
b) Ocean Area Regions AOR-E, AOR-W, POR or IOR.
c) Sea Area A2.
d) Coastal and Inland Waters.

65. Vessels operating in which sea area(s) are required to carry either Inmarsat or HF equipment or a
combination thereof under GMDSS?
a) All sea areas.
b) A3
c) A4
d) A1

***Updated by: 2/M LINO A. ARRIESGADO


Date: OCT. 10, 2018
47
66. Which of the following is a functional or carriage requirement for compulsory vessels?
a) A compulsory vessel must carry at least two (2) licensed GMDSS Radio Operators.
b) A compulsory vessel must satisfy certain equipment carriage requirements that are determined
by where the vessel sails.
c) A compulsory vessel must be able to transmit and respond to distress alerts.
d) All of the above.

67. Which communications functions must all vessels be capable of performing under GMDSS as defined
by the International Maritime Organization?
a) Radio Direction Finding.
b) Distress alerting to and from vessels, search and rescue coordination, on-scene communications,
signals for locating, maritime safety information, general and bridge-to-bridge communications.
c) Communications in each of the operational ocean areas.
d) All communications possible within the International Safety-Net service.

68. GMDSS-equipped ships will be required to perform which of the following communications functions?
a) Distress alerting and maritime safety information.
b) Search and Rescue coordination and on-scene communications.
c) Bridge-to-bridge and general radio communications.
d) All of these

69. What equipment can be used to receive Maritime Safety Information?


a) Navtex.
b) EGC receiver.
c) HF NBDP.
d) All of the above.

70. Which of the following is a required GMDSS function?


a) Bridge-to-Bridge communications.
b) Reception of weather map facsimile broadcasts.
c) Both of the above.
d) None of the above.

71. Which of the following is a required GMDSS function?


a) Transmit and receive locating signals.
b) Transmit and receive general communications.
c) Both of the above.
d) None of the above.

***Updated by: 2/M LINO A. ARRIESGADO


Date: OCT. 10, 2018
48
72. Within a single sea area, what is the primary reason GMDSS imposes carriage requirements for different
radio subsystems?
a) Redundancy in duplicating all operational functions in the event of a system failure.
b) Each subsystem has a specific purpose and capabilities that generally cannot be duplicated by
other subsystems.
c) Different radio systems may be used by the various authorities.
d) The ability to communicate in all modes with any of the shore stations.

73. If operating within Ocean Area A1, and outside of NAVTEX coverage, a GMDSS-equipped vessel must
carry?
a) An Inmarsat-A terminal.
b) A GPS receiver.
c) Equipment capable of maintaining a continuous DSC watch on 2187.5 kHz.
d) Equipment capable of reception of maritime safety information by the Inmarsat enhanced group
call system,or HF NBDP.

74. What is the equipment carriage requirement for survival craft under GMDSS?
a) At least three approved two-way VHF radiotelephones on every passenger ship and cargo ships
of 500 gross tons and upwards.
b) At least two approved two-way VHF radiotelephones on every cargo ship between 300-500
gross tons.
c) At least one radar transponder must be carried on every cargo ship of 300-500 gross tons and
two transponders (one for each side) of every passenger ship and every cargo ship of 500 gross
tons and upward.
d) All of these.

75. What is the MID?


a) Mobile Identification Number.
b) Maritime Identification Digits.
c) Marine Indemnity Directory.
d) Mobile Interference Digits.

76. What does the MID (Maritime Identification Digits) signify?


a) Port of registry.
b) Nationality.
c) Gross tonnage.
d) Passenger vessel.

***Updated by: 2/M LINO A. ARRIESGADO


Date: OCT. 10, 2018
49
77. Which CES should a GMDSS Radio Operator select if his/her vessel is off the Pacific Coast of the
United States?
a) 002 Goonhilly.
b) 004 Eik
c) 001 Santa Paula.
d) Rogaland Radio

78. Which CES should a GMDSS Radio Operator select if his/her vessel is off the Atlantic Coast of the
United States?
a) 001 Southbury
b) 002 Goonhilly
c) 004 Eik
d) Rogaland Radio

79. Which Earth Station would a vessel be utilizing if operating off the Pacific Coast of the United States?
a) 002 Goonhilly.
b) 001 Southbury.
c) 001 Santa Paula.
d) KPH (San Francisco, CA).

80. Which Earth Station would a vessel be utilizing if operating off the Atlantic Coast of the United States?
a) 001 Southbury.
b) 001 Santa Paula.
c) WCC (Chatham, MA).
d) 027 Odessa.

81. Which operator’s certificate gives you authority to sail on any GMDSS ship in all sea areas?
a) GOC
b) ROC
c) MOC
d) DSC

82. What is the name of the international convention that regulates GMDSS?
a) STCW
b) MMSI
c) SOLAS
d) GMDSS

***Updated by: 2/M LINO A. ARRIESGADO


Date: OCT. 10, 2018
50
83. The prowords which means “ I have received and understood your message and have nothing more to
transmit “ and reply is not expected is:
a) Roger, again
b) Roger, over
c) Roger, say again
d) Roger, out

84. What is the minimum requirement of a GMDSS radio operator?


a) Marine Radio Operator Permit and GMDSS Endorsement.
b) General Radiotelephone Operator license and Radar endorsement.
c) GMDSS Radio Operator license.
d) General Radiotelephone license or First or Second Class Radiotelegraph license with GMDSS
Radio Maintainer's endorsement.

85. Who has ultimate in control of service at a ship’s radio station?


a) The Master of the Ship
b) A holder of a First Class Radiotelegraph Certificate with six months service
endorsement
c) The Radio-Officer-in-Charge authority by the captain of the vessel
d) An appointed licensed radio operator who agrees to comply with all radio
regulations in force.

***Updated by: 2/M LINO A. ARRIESGADO


Date: OCT. 10, 2018
51
GOC FOR GMDSS REVIEWER
COMPETENCE 2 Provide Radio Services in Emergencies (GOC)
KUP1 – ABANDON SHIP

1. Which action should the GMDSS radio operator take in a distress situation when embarking in survival
craft?
a) Switch on EPIRB and SART immediately and leave on.
b) EPIRB and SART switched on manually prior to embarking; remain aboard vessel in distress.
c) Notify RCC (Rescue Coordination Center) through VHF DSC in portable equipment.
d) Communicate via Inmarsat-C from the survival craft.

2. What equipment is used to provide the means for locating a survival craft or mother ship in distress?
a) EPIRB
b) RDF
c) SART
d) EPIRB and SART

3. What can be defined as Radio Transmissions intended to facilitate the finding of a mobile unit in distress
of the location of survivors?
a) Alerting Signal
b) Radiowave propagation
c) Locating Signal
d) 0.44 second burst from the EPIRB

4. What can be defined as Radio Transmissions intended to facilitate the finding of a mobile unit in distress
of the location of survivors?
a) Homing Signal
b) Distress Signal
c) Locating Signal
d) Satellite Signal

5. When will the float-free Emergency Position Indicating Radio Beacon be activated after abandoning
ship?
a) Immediately after floating free
b) When manually activated
c) Automatic Operation
d) When out of the bracket

***Updated by: 2/M LINO A. ARRIESGADO


Date: OCT. 10, 2018
52
6. When should the emergency position indicating radio beacon be activated after abandoning the vessel?
a) Only when another vessel is in sight
b) It will automatically start to transmit when the EPIRB is out of the bracket and deployed into
water.
c) Only if you flash with water
d) Immediately

7. If a ships sinks, what device is designed to float free of the mother ship, is turned on automatically and
transmit a distress signal?
a) EPIRB on 121.5 MHz/243 MHz or 406.025 MHz.
b) EPIRB on 2182 kHz and 405.025 kHz.
c) Bridge-to-bridge transmitter on 2182 kHz.
d) Auto alarm keyer on any frequency.

8. Which device provides the main means in the GMDSS for locating ships in distress, or their survival
craft?
a) Radio Direction Finder.
b) Satellite EPIRBs.
c) MF/HF DSC.
d) VHF homing device.

9. What part of a satellite EPIRB may function as a visual aid to rescue vessels?
a) A 121.5 MHz emergency transmitter in a satellite EPIRB.
b) Strobe light.
c) 406 MHz signal from a satellite EPIRB.
d) Loud beeping tone emitted by the unit once activated.

10. Which of the following EPIRBs is most likely to be used to transmit a distress alert signal?
a) S-Band EPIRBs.
b) 406 MHz EPIRBs.
c) Class A EPIRBs.
d) 121.5/243 MHz EPIRBs.

11. What information is transmitted by a 406 MHz EPIRB alert?


a) Vessel position and nature of distress.
b) A unique Hexadecimal I.D. number.
c) Vessel name and identification.
d) None of the above.

12. A distress signal transmitted from which EPIRB is relayed by an INMARSAT satellite.
a) Class A EPIRBs.
b) Class B EPIRBs.
c) L-band EPIRBs on the designated frequency.
d) 406 MHz EPIRBs broadcast to all vessels for relay to a CES.
***Updated by: 2/M LINO A. ARRIESGADO
Date: OCT. 10, 2018
53
13. Which of the following statements concerning satellite EPIRBs is true?
a) Once activated, these EPIRBs transmit a signal for use in identifying the vessel and for
determining the position of the beacon.
b) The coded signal identifies the nature of the distress situation.
c) The coded signal only identifies the vessel's name and port of registry.
d) If the GMDSS Radio Operator does not program the EPIRB, it will transmit default information
such as the follow-on communications frequency and mode.

14. Which EPIRB transmits a distress alert that is received and relayed by an INMARSAT satellite?
a) Class A EPIRBs.
b) Class B EPIRBs.
c) L-band EPIRBs.
d) Category I EPIRBs.

15. What is the quickest method of transmitting a DSC distress alert?


a) Press the “Distress Hot Key”.
b) Make a “MAYDAY” call on Ch-70.
c) Make a “MAYDAY” call on Ch-16.
d) Select “Distress” priority from the menu.

16. Which equipment may not be utilized to provide locating signals in GMDSS?
a) SART’s signal from Float-Free transponder
b) 406 EPIRB
c) Loran C or GPS signal
d) Satellite or Float-Free EPIRB’s , 9 Ghz signal SART’s

17. Which equipment may NOT be utilize to provide locating signal in GMDSS?
a) Float-Free EPIRB
b) 3 Ghz signals of radar transponder
c) Both choices are correct
d) Neither choices is correct

18. What is an example of a locating signal?


a) SSB Phone traffic
b) Ship to shore transmission
c) Loran C
d) A float- free EPIRB

19. When the EPIRB is out of the cradle or release from its bracket will ____ transmit when deployed into
the water:
a) Automatically
b) Immediately
c) Start
d) not
***Updated by: 2/M LINO A. ARRIESGADO
Date: OCT. 10, 2018
54
20. What is the purpose of the SART's audible tone alarm?
a) It informs survivors that assistance may be nearby.
b) It informs survivors when the battery's charge condition has weakened.
c) It informs survivors when the SART switches to the "standby" mode.
d) It informs survivors that a nearby vessel is signaling on DSC.

21. How can SART’s effective range be maximized?


a) The SART should be placed in water immediately upon activation.
b) The SART should be held as high as possible.
c) Switch the SART into the "high" power position.
d) If possible, the SART should be mounted horizontally so that its signal matches that of the
searching radar signal.

22. In a lifeboat or liferaft, what is a method of maximizing the effectiveness of an SART?


a) Place the SART into the sea as soon as possible to begin transmitting.
b) Hold or mount the unit as high as possible.
c) Extend the length of the transmitting antenna.
d) Replace the internal battery with the AC power adapter.

23. Why must the SART be mounted as high as possible in the survival craft?
a) To increase the range
b) To be seen better on the radar
c) Because it increases the power
d) So that it will not get wet

24. What is distress traffic?


a) In radiotelegraphy, SOS sent as a single character, in radiotelephony, speaking of the word”
MAYDAY”
b) Health and welfare messages concerning the immediate protection of property and safety of
human life.
c) Internationality recognized communications relating to emergency, situations.
d) All messages relative to the immediate assistance required a ship, air craft or other vehicle in
imminent danger.

25. What are the highest priority communications from ships at sea?
a) All critical messages traffic authorized by the ship’s master.
b) Navigation and meteorological warnings.
c) Distress calls and communications preceded by the international urgency and safety signals.
d) Authorized government communications for which priority right has been claimed,

***Updated by: 2/M LINO A. ARRIESGADO


Date: OCT. 10, 2018
55
26. Your ship received a distress relay on DSC VHF CHANNEL 70, ON what channel would you reply?
a) 70
b) 06
c) 13
d) 16

27. Under what conditions would you relay a DSC distress alert?
a) If the mobile unit in distress is incapable offurher distress alert communications.
b) If no coast stations mobile unit acknowledgement is observed.
c) A and B are both possible
d) You should never relay such an alert the Coast station and RCC will do that.

28. In radiotelephony “SEELONCE FEENEE” means?


a) Distress has not ended and normal working may not be resumed.
b) Distress has ended and normal working may be resumed.
c) Restricted may not be resumed.
d) Search and rescue operation is still on going.

29. Message 1 in the DSC distress call contains the?


a) Time indications (UTC) when the coordinates were valid.
b) Single character to indicates the types of communications which is preferred by the station in
distress for subsequent exchange of distress traffic.
c) Distress coordinates messages.
d) Nature of Distress

30. The distress relay alert shall contain the following except:?
a) Position
b) Port of departure and destination of the mobile unit in distress.
c) Time
d) Nature of Distress

31. The distress alerts that used to alert other ships in the vicinity of the ship in distress and are based on the
used on DSC IN THE VHF AND MF Bands.?
a) Ship-to ship distress alerts
b) Ships to shore distress alert
c) Shore to ship distress alert
d) Ships to aircraft distress alert.

32. The MMSI is used in all of the following DSC calls except?
a) Routine alert
b) Urgency alert
c) Safety alert
d) Distress calls and all ship’s call

***Updated by: 2/M LINO A. ARRIESGADO


Date: OCT. 10, 2018
56
33. The MMSI number plan is used in all of the following GMDSS equipment EXCEPT…
a) VHFDSC, MFDSC, HFDSC and radio telex
b) Inmarsat-C
c) Inmarsat and COSPAS/SARSAT EPIRBs
d) SART

34. The mechanism that allows a Float-free EPIRB is;


a) Hydraulic Release Mechanism
b) Spring Release Mechanism
c) Hydrostatic Release Mechanism
d) Air Release Mechanism

35. Which piece of required GMDSS equipment is the primary source of transmitting locating signals?
a) Radio Direction Finder (RDF).
b) An EPIRB transmitting on 406 MHz.
c) Survival Craft Transceiver.
d) A SART transmitting on 406 MHz.

36. Which communication functions must all vessels be capable of performing under GMDSS as defined by
international maritime organization?
a) Ship to ship alerting
b) Ship to shore alerting
c) Shore to Ship alerting
d) Distress alerting to and from vessels, search and rescue coordination, on-scene communication,
signals for locating, maritime safety information, general and bridge-to-bridge communications

37. What is the meaning of the term SART?


a) The U.S. Coast Guard Coordinated Search and Rescue Team
b) The COSPAS/SARSAT System
c) The Search and Rescue Team coordinated by a National Rescue Coordination Center (RCC)
d) Search and Rescue Radar Transponder

38. Equipment which can be utilized to provide locating signals in GMDSS?


a) Loran C, or GPS signals
b) Radio Direction Finder
c) EPIRB
d) SART

39. SARSAT stands for:


a) Search and Rescue Satellite Aided Tracking
b) Coast Earth Station Search and Rescue Aided Tracking
c) Committee on Search and Rescue Aided Tracking
d) Search and Rescue Satellite Alternating Tracking

***Updated by: 2/M LINO A. ARRIESGADO


Date: OCT. 10, 2018
57
40. What may be used as a homing signal by the search and rescue vessels in the immediate vicinity of the
ship in distress?
a) Flare gun.
b) Strobe Light.
c) A 121.5 MHz emergency transmitter in a satellite EPIRB.
d) 406 MHz signal from a satellite EPIRB.

41. What equipment is used in or near the survival craft?


a) Navtex.
b) Fathometer.
c) Cospas-Sarsat.
d) EPIRB.

42. Which of the following would best be used for visual detection of a distressed vessel?
a) A 9-GHz SART's beacon.
b) An EPIRB's strobe light.
c) A 121.5-MHz EPIRB beacon.
d) A 406-MHz EPIRB beacon.

43. The GMDSS role performed by EPIRB is as?


a) primary method of alerting
b) to send MMSI number
c) Secondary means of distress alerting
d) to participate in SAR operation

44. Which statement is NOT true regarding the SART?


a) Responds to interrogations by a vessel's X-Band radar.
b) This is a 6 GHz transponder capable of being received by a vessel's X-band navigational radar
system.
c) This is a 9 GHz transponder capable of being received by another vessel's X-band navigational
radar system.
d) Transmits a distinctive signal for easy recognition.

45. Which statement is NOT true regarding the SART?


a) Responds to interrogations by a vessel's X-Band radar.
b) Transmits on the 9 GHz band reserved for navigational radar.
c) Operates in conjunction with a vessel's S-Band radar.
d) Transmits a distinctive code for easy recognition.

***Updated by: 2/M LINO A. ARRIESGADO


Date: OCT. 10, 2018
58
46. A distress signal:
a) consists of 5 or more short blasts of the fog signal apparatus
b) consists of the raising and lowering of a large white flag
c) May be used individually or in conjunction with other distress signals
d) is used to indicate doubt about another vessel's intentions

47. All of the following are recognized distress signals under the Navigation Rules EXCEPT:
a) A green star signal
b) orange-colored smoke
c) red flares
d) the repeated raising and lowering of outstretched arms

48. An aircraft has indicated that he wants you to change course and follow him. You cannot comply
because of an emergency on board.What signal should you make?
a) Fire a red flare at night or a red smoke signal by day
b) Send the Morse signal "N“ by flashing light
c) Make a round turn (360 degrees) and resume course
d) Make an "S" turn (hard right then hard left) and resume course

49. At sea red rocket signals are seen. This is not reported by radio. You have to begin the distress alert
procedure via VHF with the term:
a) MAYDAY RECU
b) MAYDAY
c) MAYDAY RELAY
d) None of the above

50. Distress alert initiated for other than an appropriate test, by communications equipment intended for
alerting, when no distress situation actually exists.
a) False alarm
b) False alert
c) False Sequence
d) False Relay

51. Distress received from any source, including communications equipment intended for alerting, when no
distress situation actually exists, and a notification of distress should not have resulted.
a) False alert
b) False alarm
c) False Sequence
d) False Relay

***Updated by: 2/M LINO A. ARRIESGADO


Date: OCT. 10, 2018
59
52. During a training exercise a submarine indicating that a torpedo has been fired will send up smoke from
a float. The smoke's color will be:
a) Black
b) red
c) orange
d) yellow

53. How should signal flares be used after you have abandoned ship and are adrift in a liferaft?
a) Immediately use all the signals at once.
b) Use all the signals during the first night.
c) Employ a signal every hour after abandoning ship until they are gone.
d) Use them only when you are aware of a ship or plane in the area.

54. By day, the signal meaning, "This is the best place to land" is a:
a) vertical motion of a red flag
b) Vertical motion of a white flag or the arms
c) white smoke signal
d) white star rocket

55. How is a distress priority message ordinarily initiated on board the vessel?
a) By dialing the correct code on the telephone remote unit.
b) By pressing one or more dedicated "distress key/s" on the equipment.
c) By contacting the CES operator, and announcing a distress condition is in existence.
d) By contacting the CES operator using the radiotelephone distress procedure "Mayday... etc.

56. What equipment is programmed to initiate transmission of distress alerts and calls to individual
stations?
a) Navtex.
b) GPS.
c) DSC controller.
d) Scanning Watch Receiver.

57. Which of the following satellite systems is of particular importance to search and rescue missions under
GMDSS?
a) AMSAT.
b) NASA/Arienne.
c) COSPAS/SARSAT.
d) COMSAT.

58. Which is the key part of the search and rescue system under GMDSS?
a) AMSAT satellites.
b) NASA satellites.
c) US Space Agency satellites.
d) COSPAS/SARSAT satellites.
***Updated by: 2/M LINO A. ARRIESGADO
Date: OCT. 10, 2018
60
59. What is the action that a GMDSS Radio Operator should take when a DSC distress alert is received?
a) No action is necessary, as the DSC control unit will automatically switch to the NBDP follow-on
communications frequency.
b) The Operator should immediately set continuous watch on the radiotelephone frequency that is
associated with frequency band on which the distress alert was received.
c) The Operator should immediately set continuous watch on VHF channel 70.
d) The Operator should immediately set continuous watch on the NBDP frequency that is
associated with frequency band on which the distress alert was received.

60. In all cases, the transmit frequency of a MF/HF console DSC distress alert:
a) Will go out first on 2187.5 kHz.
b) Will go out on 8 MHz and 2 MHz and one other DSC distress frequency.
c) It depends upon operator DSC Call set up entries.
d) None of the above.

61. What is the proper procedure to be followed upon receipt of a distress alert transmitted by use of Digital
Selective Calling techniques?
a) Set watch on the DSC alerting frequency in the band of frequencies the alert was received.
b) Set watch on the radiotelephone distress and safety frequency associated with the distress and
safety calling frequency on which the distress alert was received.
c) Set a continuous watch on VHF-FM Channel 13, 16 and DSC on Channel 70.
d) Ship stations equipped with narrow-band direct-printing equipment should respond to the
distress alert as soon as practicable by this means.

62. DSC is used primarily to:


a) Receive weather warnings, navigational notices and other marine safety information.
b) Provide routine communications with the ship owner.
c) Transmit and receive distress, urgent and safety alerts to and from other ships and shore stations
via radio.
d) Report ship's position to search-and-rescue authorities via satellite.

63. Equipment for radiotelephony use in survival craft stations under GMDSS must have what capability?
a) Operation on Ch-16.
b) Operation on 457.525 MHz.
c) Operation on 121.5 MHz.
d) Any one of these.

64. Equipment for radiotelephony use in survival craft stations under GMDSS must have what
characteristic(s)?
a) Operation on Ch-16.
b) Watertight.
c) Permanently-affixed antenna.
d) All of these.

***Updated by: 2/M LINO A. ARRIESGADO


Date: OCT. 10, 2018
61
65. Which of the following has been designated for “On-scene” communications in GMDSS?
a) Ch-24
b) Ch-2182
c) Ch-70
d) Ch-16 on VHF radiotelephone and 2174.5 kHz using MF SITOR.

66. Which of the following channels is designated as the VHF follow-on communications channel and is
required in all portable survival craft equipment?
a) Ch-6
b) Ch-13
c) Ch-16
d) Ch-70

67. For “On-scene” communications, vessels in distress and SAR Aircraft should use?
a) VHF Ch-70, 4125 kHz J3E, 5680 kHz J3E
b) VHF Ch-16, 4125 kHz J3E, 3023 kHz J3E
c) VHF Ch-16, 4125 kHz F1B, 3023 kHz J3E
d) None of the above.

68. The phases of emergency at sea established for classifying incidents and determining the actions to be
taken are the following EXCEPT:
a) uncertainty phase
b) alert phase
c) distress phase
d) Critical phase

69. When positive information is received that a ship or a person on board is in grave and imminent danger
and in need of immediate assistance, the emergency phase declared is:
a) uncertainty phase
b) alert phase
c) Distress phase
d) critical phase

70. When a ship has been reported overdue at destination or it has failed to make an expected position or
safety report, the emergency phase declared is:
a) Uncertainty phase
b) alert phase
c) distress phase
d) critical phase

***Updated by: 2/M LINO A. ARRIESGADO


Date: OCT. 10, 2018
62
71. When there is apprehension regarding the safety of a ship or the persons on board or information has
been received indicating that the operational efficiency of a ship is impaired but not to the extent that a
distress situation is likely, the emergency phase declared is:
a) uncertainty phase
b) Alert phase
c) distress phase
d) critical phase

72. “On-scene" communications would best be represented by?


a) NBDP on 2174.5khz
b) Sending DSC alert on VHF Ch-70.
c) Using Inmarsat-C "hot-key" function.
d) None of the above.

73. You have abandoned ship and after two days in a liferaft you can see an aircraft near the horizon
apparently carrying out a search pattern. You should __________.
a) Use visual distress signals in conjunction with the EPIRB
b) Use the voice transmission capability of the EPIRB to guide the aircraft to your raft
c) Switch the EPIRB to the homing signal mode
d) Turn on the strobe light on the top of the EPIRB

74. To make an Emergency Call when using Inmarsat Fleet Broadband telephone services, you should dial:
a) 000
b) 101
c) 505
d) 999

***Updated by: 2/M LINO A. ARRIESGADO


Date: OCT. 10, 2018
63
GOC FOR GMDSS REVIEWER
COMPETENCE 2 Provide Radio Services in Emergencies (GOC)
KUP2 – FIRE ON BOARD SHIP

1. Channel 70 for digital selective calling for Distress, Safety and Calling frequency operate at:
a) 158.526 MHz
b) 156.526 MHz
c) 157.526 MHz
d) 159.526 MHz

2.What is the channel used for distress call such as fire on board?.
a) Channel 16
b) Channel 70
c) Channel 13
d) Channel 69

3.What is priority will you use in sending your message by fax, if your ship in or fire?
a) Urgent
b) Safety
c) Distress
d) Emergency

4.Which statement concerning GMDSS distress alerts, such as fire on board is TRUE?
a) Information contained in a distress alerts includes the name and position of the
distressed vessel, and may include additional information such as nature of distress and
what kind of assistance that may be required.
b) Ship to shore distress alerts are used to alert other ships in port of navigational hazards.
c) Ship-to-ship distress alerts are used to alert other ships in the vicinity of navigational hazards and
bad weather.
d) The vessel nearest to the emergency must notify the Coast Guard before leaving the vicinity.

5.What is usually the first step for a GMDSS Radio Operator to take when initiating a distress priority
message, such as fire onboard, via INMARSAT?
a) By dialing the correct code on the telephone remote unit.
b) By pressing a "Distress Button" or "Distress Hot Key(s)" on the equipment.
c) By contacting the CES operator and announcing a distress condition is in existence.
d) By contacting the CES operator using the radiotelephone distress procedure "Mayday"... etc.

***Updated by: 2/M LINO A. ARRIESGADO


Date: OCT. 10, 2018
64
6.In the event of fire onboard why is it important to transmit MAYDAY or PAN PAN signals, even if not in
Distress?
a) To alerts possible future distress call.
b) To warn the other ship involved in the collision.
c) To attract attention of the rescuer
d) To alert the coast guards

7.When will be the float-free Emergency Position Indicating Radio Beacon be activated after abandoning
ship due to fire onboard?
a) When manually activated
b) Automatic Operation
c) When out of the bracket
d) Immediately after floating free

8.You are approaching another vessel and see that she has a signal flag J hoisted. What should you do?
a) Keep well clear of the vessel because she is on fire and has dangerous cargo onboard or she is
leaking dangerous cargo.
b) Approach the vessel and give assistance
c) Try to communicate the vessel
d) No action to be done

9.A vessel is on fire and require immediate assistance. The VHF call starts with:
a) “MAYDAY” spoken 3 times
b) MAYDAY (3x)
c) PAN PAN PAN (3x)
d) SECURITE (3x)

10.The VHF radiotelephone calling, safety, distress, frequency is ________.


a) 156.300 MHz (Channel 06)
b) 156.650 MHz (Channel 13)
c) 156.800 MHz (Channel 16)
d) 156.525 MHz (Channel 70)

11.You should send a DSC- message because of a fire on board and assistance by other ships required have to
choice the category?
a) Urgency
b) Safety
c) Emergency
d) Distress

***Updated by: 2/M LINO A. ARRIESGADO


Date: OCT. 10, 2018
65
12. Which of the following frequencies is normally used for distress and safety communications?
a) 490 kHz
b) 518 kHz
c) 4209.5 kHz
d) 2174.5 kHz

13. Which channel is designated for GMDSS Digital Selective Calling?


a) Ch-06
b) Ch-13
c) Ch-16
d) Ch-70

14, When operating in coastal waters (sea area A1), a GMDSS-equipped vessel must:
a) Maintain a continuous DSC watch on 8514.5 kHz.
b) Maintain a continuous aural watch on 2182 kHz.
c) Maintain a continuous DSC watch on VHF channel 16.
d) Maintain a continuous DSC watch on VHF channel 70.

15. Which statement concerning Distress Alert such as fire on board is true?
a) To request immediate assistance to ship owner and coast guard
b) A distress maybe use to alert other vessel and authorities in port
c) To alert the rescuers
d) To contact the RCC immediately

16.What is the spoken emergency signal for a distress signal over a VHF radio?
a) “MAYDAY “ MAYDAY” spoken 3 times
b) “SOS” spoken 3 times
c) “MAYDAY” spoken 3 times
d) “SOS” “SOS” spoken 3 times

17.Which of the following channels and modes should be used when initiating a distress alert transmission?
a) Ch-6 DSC.
b) Ch-6 Radiotelephony.
c) Ch-13 Radiotelephony and Ch-16 DSC.
d) Ch-70 DSC.

18, In GMDSS, the channel used for VHF DSC calling for distress and safety is:
a) Channel 12
b) Channel 13
c) Channel 16
d) Channel 70

***Updated by: 2/M LINO A. ARRIESGADO


Date: OCT. 10, 2018
66
19, In GMDSS, the channel used for VHF DSC calling for public correspondence is:
a) Channel 12
b) Channel 13
c) Channel 16
d) Channel 70

20, How is a distress message, such a fire onboard normally initiated through Inmarsat?
a) Pressing a distress key on the equipment
b) Transmit distress alert by any means
c) Send DSC manually
d) Send alert automatically

21, If your vessel is equipped with a SSB radio, what frequency would you use to initiate a distress call such
as fire onboard?
a) 2174.5 KHz
b) 2177.0 KHz
c) 2182.0 KHz
d) 2189.5 KHz

22, The Radiotelephone Urgency signal is:


a) Mayday
b) Pan Pan
c) Securite
d) Seelonce Feenee

23, What is the Internationally recognized “ URGENCY” signal?


a) The letters “TTT” transmitted three times by radiotelegraphy.
b) Three oral repetitions of the word “safety” sent before the call.
c) The word “PAN PAN” spoken three times before the urgent call.
d) The pronouncement of the word “Mayday”.

24, Ship-to-ship distress alerting is primarily based on:


a) Satellite Communication
b) Mobile Telephone
c) DSC in the VHF and MF Bands
d) VHF Telephony

25, Which of the following watches must a compulsory vessel maintain when sailing in Sea Area A1?
a) A continuous DSC watch on 8414.5 kHz plus one other HF DSC frequency.
b) A continuous DSC watch on 2187.5 kHz.
c) A continuous DSC watch on Ch-16.
d) A continuous DSC watch on Ch-70.

***Updated by: 2/M LINO A. ARRIESGADO


Date: OCT. 10, 2018
67
26, Which of the following are the MF/HF DSC Distress watch frequencies
a) 2177.5, 4210.0, 6314.0, 8416.5 12579.0, 16806.5
b) 2182.0, 4125.0, 6215.0, 8291.0, 12290.0, 16420.0
c) 2187.5, 4207.5, 6312.0, 8414.5, 12577.0, 16804.5
d) 2174.5, 4177.5, 6268.0, 8376.5, 12520.0, 16695.0

27, What is the proper format for a distress follow on voice transmission? (3x is three times),
a) All Ships 3x this is Ship's Name/Call Sign 3x Mayday Position.
b) Mayday 3x this is Ship's Name/Call Sign 3x Distress category.
c) Both of the above.
d) None of the above.

28, What information should be included in a distress follow on voice transmission?


a) Ship's Name and Call Sign.
b) Ship's position.
c) Ship's MMSI number.
d) All of the above.

29, What are the operation of emergency equipment & application of emergency procedures?
a) Distress signal
b) Routine signal
c) Safety Signal
d) Urgency Signal

30, The correct DSC Alert category to be used when your own vessel is in grave and imminent danger is:
a) Distress
b) Urgency
c) Mayday
d) Pan Pan

31, The correct DSC Alert, Call or Announcement to be used before sending a Distress Message on behalf
of another vessel is:
a) MAYDAY RELAY
b) Distress Alert Relay
c) Distress
d) Urgency

32, A man aboard a vessel, signaling by raising and lowering his outstretched arms to each side, is
indicating_______.?
a) danger, stay away
b) all is clear, it is safe to pass
c) A distress signal
d) all is clear, it is safe to approach

***Updated by: 2/M LINO A. ARRIESGADO


Date: OCT. 10, 2018
68
33, What action should you take after sending a false distress alert on Inmarsat–C?
a) Press the “Distress” hot keys then press the “cancel” key.
b) Select “transmit” menu and send a cancel message via the CES used for the distress alert.
c) Both of the above
d) None of the above

34, In Morse Code signaling by Hand Flag or Arms, following are the transmitting signals to a receiving
station except for;?
a) To call up a station
b) A receiving station – in answer
c) Completing the signal
d) Sending the distress message

35, What is the proper procedure to be followed upon receipt of a distress alert transmitted by use of Digital
Selective Calling Techniques?
a) Set watch on the DSC alerting frequency in the band of frequencies the alert was received.
b) Set watch on the radiotelephone distress and safety frequency associated with the distress and
safety calling frequency on which the distress alert was received.
c) Set a continuous watch on VHF-FM Channel 13, 16 and DSC on Channel 70.
d) Ship stations equipped with narrow band direct printing equipment should respond to the distress
alert as soon as practicable by this means.

36, What is the meaning of “Reserve Source of Energy”?


a) High caloric value items for lifeboat, per SOLAS regulations.
b) Diesel fuel stored for the purpose of operating the powered survival craft for a period equal to or
exceeding the U.S.C.G. and SOLAS requirements.
c) Power to operate the radio installation and conduct Distress and Safety communications in the
event of failure of the ship's main and emergency sources of electrical power.
d) The diesel fueled emergency generator that supplies AC to the vessel’s Emergency power bus.

37, A communications network that uses a combination of line facilities, i.e., trunks, loops, or links, some of
which use only analog or quasi-analog signals and some of which use only digital signals, system for
Long Range Resource Tracking in Search and Rescue Scenarios.
a) Hybrid (communication networks)
b) Internet
c) Fiber
d) Wi-Fi

***Updated by: 2/M LINO A. ARRIESGADO


Date: OCT. 10, 2018
69
38, What are the advantages of Digital Communications over Hybrid or Analogue Communications?
a) Covers lesser distances
b) Covers minimum distances
c) Covers high distances
d) Covers greater distances (quantized carry more informations in a seconds, noise immunity,
multiplexing, easy to store, resistant to additive noises, used for long distance and transmission
error can be detected easily. )

39, What is an Analog Computer?


a) Separate
b) Counting
c) Continuous
d) Resistive

40, What is a Continuous Computer Paper?


a) Clean Paper
b) Dot Matrix
c) Highest quality of cleanliness
d) Required carbon

41, What is a Hybrid Computer?


a) Having a Digital and Analog features
b) Modern
c) Counting
d) Analogue

42, What Computer is flexible?


a) Supper Computer
b) Large Computer
c) Personal Computer
d) Fabric Computer
43, A GMDSS equipped ship has to send a distress call. The call sequence to be followed.
a) send DSC distress relay alert – wait for acknowledgement – send the distress message
b) Send DSC distress alert - wait for acknowledgement send the distress message
c) send DSC distress alert - send the distress message - wait for acknowledgement
d) send DSC distress alert - send the distress message without waiting for acknowledgement

***Updated by: 2/M LINO A. ARRIESGADO


Date: OCT. 10, 2018
70
44, In sea areas A3 and A4, the transmission of DSC distress alert on HF band should be addressed to:
a) Coast Stations
b) All Stations
c) Ships in a certain geographical area
d) Selected Ship Stations

45, You are onboard a ship in sea area A3 and you have received a DSC distress alert on 8414.5 KHz. If no
DSC distress acknowledgement is received from a coast station within 3 minutes, you should:
a) disregard the distress alert
b) acknowledge the distress alert by DSC
c) Transmit a DSC distress relay alert
d) acknowledge the distress alert by R/T

46, Acknowledgement of a DSC distress alert by use of DSC is normally made by:
a) Coast station only
b) any station which heard it
c) all stations which heard it
d) RCC only

47, How many frequencies are available under GMDSS for DSC distress related calls?
a) 4
b) 7
c) 5
d) 6

48, How many HF frequencies are available for DSC distress related calls?
a) 7
b) 5
c) 4
d) 3

49, You have receive a DSC distress alert on 2187.5 KHz and it is indicated that the distress message is to
be transmitted by SSB telephony. Which frequency are you supposed to listen to?
a) 2174.5 KHz
b) 2177.0 KHz
c) 2182.0 KHz
d) 2189.5 KHz

50, The DSC distress alerting frequency in the MF band is:


a) 2182.0 KHz
b) 2177.0 KHz
c) 2185.5 KHz
d) 2187.5 KHz

***Updated by: 2/M LINO A. ARRIESGADO


Date: OCT. 10, 2018
71
51, The kind of emission classes which can be used on 2182 KHz is any of the following EXCEPT:
a) G3E
b) A3E
c) H3E
d) J3E

52, Safety communication between ships from the position from which the ships are normally navigated is
called:
a) general communication
b) on-scene communication
c) public correspondence
d) Bridge-to-bridge communication

53, The distress alerts used to alert Rescue Coordination Centers via coast stations or coast earth stations
that a ship is in distress is:
a) Ship-to-shore distress alerts
b) ship-to-ship distress alerts
c) shore-to-ship distress alerts
d) shore-to-shore distress alerts

54, The distress alerts which are based on the use of transmissions via satellites (from a ship earth station or
a satellite EPIRB) and terrestrial services (from ship stations and EPIRBs) are:
a) ship-to-ship distress alerts
b) Ship-to-shore distress alerts
c) shore-to-ship distress alerts
d) shore-to-shore distress alerts

55, The distress alerts used to alert other ships in the vicinity of the ship in distress and are based on the use
of DSC in the VHF and MF bands:
a) ship-to-shore distress alerts
b) shore-to-ship distress alerts
c) shore-to-shore distress alerts
d) Ship-to-ship distress alerts

56, A shore-to-ship distress alert relay initiated by a station or a Rescue Coordination Centre which receives
a distress alert shall be addressed to any of the following EXCEPT:
a) Coast station
b) all ships
c) selected group of ships
d) specific ship

***Updated by: 2/M LINO A. ARRIESGADO


Date: OCT. 10, 2018
72
57, The distress alert relay shall contain the following EXCEPT:
a) the identification of the mobile unit in distress
b) the position of the mobile unit in distress
c) Port of departure and destination of the mobile unit in distress
d) all other information which might facilitate rescue

58, Distress traffic consists of the following EXCEPT:


a) all messages relating to the immediate assistance required by the ship in distress
b) Medical transports messages
c) search and rescue communications
d) on scene communications

59, Communications necessary for the coordination of ships and aircraft participating in a search and rescue
operation following a distress alert and include communications between RCC’s and any OSC or CSS
in the area of the distress incident is called:
a) SAR coordinating communications
b) on-scene communications
c) general radio communications
d) bridge-to-bridge communications

60, Communications between the ship in distress and assisting units which relate to the provision of
assistance to the ship or the rescue of survivors is called:
a) SAR coordinating communications
b) general radio communications
c) bridge-to-bridge communications
d) On-scene communications

61, The communication necessary for the coordination of the ships and aircraft participating in a search
resulting from a distress incident is called:
a) On-scene communications
b) SAR coordinating communications
c) Bridge-to-bridge communication
d) Distress communication

62, In GMDSS, those communication between ship stations and shore-based communication networks
which concern the management and operation of the ship and may have an impact on its safety is, e.g.
orders for pilot and and tug services, charts replacement, repairs, etc. is called:
a) SAR coordinating communications
b) Bridge-to-bridge communications
c) On-scene communications
d) General communications

***Updated by: 2/M LINO A. ARRIESGADO


Date: OCT. 10, 2018
73
63, For a DSC distress call, the distress information is contained in four messages. Message 1 is the:
a) distress coordinates message
b) time indication (UTC) when the coordinates were valid
c) Nature of distress message
d) single character to indicate the type of communication which is preferred by the station in
distress for subsequent exchange of distress traffic

64, Message 2 in the DSC distress call contains the:


a) time indication (UTC) when the coordinates were valid
b) nature of distress message
c) Distress coordinates message
d) single character to indicate the type of communication which is preferred by the station in
distress for subsequent exchange of distress traffic

65, Message 3 in the DSC distress call contains the:


a) Time indication (UTC) when the coordinates were valid
b) nature of distress message
c) distress coordinates message
d) single character to indicate the type of communication which is preferred by the station in
distress for subsequent exchange of distress traffic

66, Message 4 in the DSC distress call contains the:


a) time indication (UTC) when the coordinates were valid
b) nature of distress message
c) distress coordinates message
d) Single character to indicate the type of communication which is preferred by the station in
distress for subsequent exchange of distress traffic

67, The class of emission designated for HF Radiotelex and DSC is:
a) F1B
b) J3E
c) A1A
d) H3E

68, The class of emission designated for all radiotelephony frequencies in the maritime HF band (3-30)MHz
is:
a) F1B
b) J3E
c) A3A
d) H3E

***Updated by: 2/M LINO A. ARRIESGADO


Date: OCT. 10, 2018
74
69, In class of emission F1B, B refers to:
a) Telegraphy, for automatic reception
b) telegraphy, for aural reception
c) telephony including sound broadcasting
d) television reception

70, In class of emission J3E, J refers to:


a) Double-Side-Band (DSB)
b) Single-Side-Band (SSB) with full carrier
c) Single-Side-Band (SSB) with reduced carrier
d) Single-Side-Band (SSB) with suppressed carrier

71, In class of emission R3E, R refers to:


a) Double-Side-Band (DSB)
b) Single-Side-Band (SSB) with full carrier
c) Single-Side-Band (SSB) with reduced carrier
d) Single-Side-Band (SSB) with suppressed carrier

72, In class of emission A3E, A refers to:


a) Double-Side-Band (DSB)
b) Single-Side-Band (SSB) with full carrier
c) Single-Side-Band (SSB) with reduced carrier
d) Single-Side-Band (SSB) with suppressed carrier

73, In class of emission H3E, H refers to:


a) Double-Side-Band (DSB)
b) Single-Side-Band (SSB) with full carrier
c) Single-Side-Band (SSB) with reduced carrier
d) Single-Side-Band (SSB) with suppressed carrier

74, Your ship is involved in a search and rescue operation and you are bothered by interference from other
stations, which signal are you going to use to impose radio silence?
a) MAYDAY RELAY
b) SEELONCE FEENEE
c) SEELONCE MAYDAY
d) SEELONCE DISTRESS

75, What element of a DSC call indicates the type of distress situation?
a) Message 1 of distress calls
b) Message 2 of distress calls
c) Message 3 of distress calls
d) Message 4 of distress calls

***Updated by: 2/M LINO A. ARRIESGADO


Date: OCT. 10, 2018
75
76, What element of a DSC call describes the emergency position with the aid of 10 digits?
a) Message 1 of distress calls
b) Message 2 of distress calls
c) Message 3 of distress calls
d) Message 4 of distress calls

77, What element of a DSC call describes the hour of the position, and given in UTC with the aid of 4
digits?
a) Message 1 of distress calls
b) Message 2 of distress calls
c) Message 3 of distress calls
d) Message 4 of distress calls

78, What element of a DSC call indicates the type of communication desired the subsequent
correspondence?
a) Message 1 of distress calls
b) Message 2 of distress calls
c) Message 3 of distress calls
d) Message 4 of distress calls

79, What element of the DSC call indicates whether one wants the communication by telephony, telex or
data transfer?
a) Message 1 of routine calls
b) Message 2 of routine calls
c) Message 3 of routine calls
d) Message 4 of routine calls

80, What element of the DSC call indicates the frequency desired for the subsequent communication?
a) Message 1 of routine calls
b) Message 2 of routine calls
c) Message 3 of routine calls
d) Message 4 of routine calls

81, What element of the DSC call which is transmitted as a control bit for the whole call?
a) End of sequence
b) Error check character
c) Phasing sequence
d) Format specifier

***Updated by: 2/M LINO A. ARRIESGADO


Date: OCT. 10, 2018
76
82, What element of the DSC call indicates if the call requires an acknowledgement of receipt and if the
current messages is a reply to another call?
a) End of sequence
b) Error check character
c) Phasing sequence
d) Format specifier

83, What element of the DSC call consists of dots sent out to enable the scanning receiver to tune itself to
the frequency (stop scanning)?
a) Dot pattern
b) Phasing sequence
c) End of sequence
d) Error check character

84, What element of the DSC call serves as the phasing-in signals preparing the receiver to receive the
information from the DSC transmitter?
a) Dot pattern
b) Phasing sequence
c) End of sequence
d) Error check character

85, You have received a DSC distress alert on HF. Your DSC receiver shows that the distress alert was
transmitted on 4207.5 kHz. On which frequency shall you set your HF receiver for reception of the
complete distress message by RT?
a) 4125 kHz
b) 4177.5 kHz
c) 4209.5 kHz
d) 4210 kHz

86, You have received a DSC distress alert on HF. Your DSC receiver shows that the distress alert was
transmitted on 6312 kHz. On which frequency shall you set your HF receiver for reception of the
complete distress message by RT?
a) 6414 kHz
b) 6312 kHz
c) 6268 kHz
d) 6215 kHz

87, You have received a DSC distress alert on HF. Your DSC receiver shows that the distress alert was
transmitted on 8414.5 kHz. On which frequency shall you set your HF receiver for reception of the
complete distress message by RT?
a) 8291 kHz
b) 8376.5 kHz
c) 8415 kHz
d) 8416.5 kHz
***Updated by: 2/M LINO A. ARRIESGADO
Date: OCT. 10, 2018
77
88, You have received a DSC distress alert on HF. Your DSC receiver shows that the distress alert was
transmitted on 12577 kHz. On which frequency shall you set your HF receiver for reception of the
complete distress message by RT?
a) 12579 kHz
b) 12577 kHz
c) 12520 kHz
d) 12290 kHz

89. A ship is in distress. How could priority be obtained over the Inmarsat Satellite System so that distress
message could be transmitted?
a) Press emergency/distress button as long as you want
b) Press emergency/distress button on and off for 5 seconds
c) Press emergency/distress button continuously for 5 seconds
d) All of the above

90. DSC controllers have a facility that allows the unit to be routinely tested without the associated
transmitter being activated. The unit should be tested while underway at least?
a) Once a day
b) Once a week
c) Twice a week
d) Once a month

91. GMDSS vessel equipped for A2, A3 or A4 must maintain a continuous DSC watch on 2187.5 khz:
a) Only in areas beyond the Inmarsat coverage
b) Only outside of areas covered by VHF-DSC
c) When directed to do so by cognizant rescue authority
d) At all times when underway

92. How many HF frequencies are available for DSC distress related Calls?
a) one
b) two
c) four
d) Five

93. How many pieces of portable GMDSS VHF transceivers must be provided for a ship of
450 gross tonnage?
a) 1
b) 2
c) 3
d) 6

***Updated by: 2/M LINO A. ARRIESGADO


Date: OCT. 10, 2018
78
94. In GMDSS, the VHF channel reserved for bridge-to-bridge communication is:
a) Channel 06
b) Channel 16
c) Channel 13
d) Channel 70

95. In the COSPAS/SARSAT system, the distress signals picked up by the satellite are place to
a ground receiving station known as:
a) LET
b) CES
c) LUT
d) CRS

96. Russian acronym which means space system for the search of vessels in distress.
a) SARSAT
b) COSPAS
c) AVARYNICH
d) None of the above

97. The distress alerts used to alert other ships in the vicinity of the ship:
a) Ship-to-shore distress alert
b) Shore-to-ship distress alert
c) Shore-to-shore distress alert
d) Ship-to-ship distress alert

98. The SART operates on 9 Ghz. What radar band does this frequency belong to:
a) C-band
b) S-band
c) L-band
d) X-band

99. The transmission speed of a DSC call on VHF band is:


a) 600 bauds
b) 1200 bauds
c) 1600 bauds
d) 2400 bauds

100, VHF channel used for communication between ships and aircraft for coordinated
SAR operations:
a) Channel 6
b) Channel 13
c) Channel 16
d) Channel 70

***Updated by: 2/M LINO A. ARRIESGADO


Date: OCT. 10, 2018
79
102, What is a distress communication?
a) An Internationally recognized communication indicating that the sender is threatened by grave
and imminent danger and request immediate assistance
b) communication indicating that the calling station has a very urgent message concerning safety
c) Radio communication which, if delayed, will adversely affect the safety of life or property
d) An official communications notification of approaching navigational or meteorological hazards

103, What is a distress traffic?


a) In radiotelegraphy, SOS sent as a single character, in radiotelephony, the speaking of the word
“MAYDAY”
b) Health and welfare messages concerning the immediate protection of property and safety of
human life
c) Internationally recognized communications relating to emergency situations
d) All messages relative to the immediate assistance required by a ship, aircraft or other vehicle in
imminent danger

104, When you received a DSC distress alert on 2187.5 khz indicating “USB telelphony” as
subsequent communication, you should listen for the distress message that follows on ____
a) 2189.5 Khz
b) 2182.0 Khz
c) 2177.0 Khz
d) 2188.0 Khz

105, Which of the following MMSI belongs to a Philippine registered vessel?


a) 248123400
b) 348123400
c) 448123400
d) 548123400

106, A valid MMSI number for a DSC call to a specific group of vessel is:
a) 3664523
b) 30327931
c) 038462941
d) 3036483

***Updated by: 2/M LINO A. ARRIESGADO


Date: OCT. 10, 2018
80
GOC FOR GMDSS REVIEWER
COMPETENCE 2 Provide Radio Services in Emergencies (GOC)
KUP3 – PARTIAL OR FULL BREKDOWN OF RADIO INSTALLATION

1. What frequency should be used to send a distress call?


a) Whatever frequency has the best chance of communicating the distress message
b) 2182 kHz
c) Only frequencies that are within your operating privileges
d) Only frequencies used by police, fire or emergency medical services

2. What is meant by CW?


a) Continuous Working (channel)
b) Continuous Wave (Morse code)
c) Continuous Watch (i.e. shore station)
d) Cohesive Wave

3. If the Watch Officer hears "PAN PAN" spoken 3 times it means:


a) A navigation or important meteorological warning should follow.
b) The station is preparing to transmit an URGENT message possibly concerning the safety of a
mobile unit or person.
c) A Mobile unit is in need of immediate assistance.
d) None of the above.

4. DSC is used primarily to ___________.


a) Receive weather warnings, navigational notices and other marine safety information.
b) Provide routine communications with the ship owner.
c) Transmit and receive distress, urgent and safety alerts to and from other ships and shore stations
via radio.
d) Report ship's position to search-and-rescue authorities via satellite.

5. What is a practical way to avoid harmful interference on an appreciately clear frequency before calling
CQ on CW or phone?
a) Send "QRL?" on CW, followed by your call sign; or, if using phone, ask if the frequency is in
use, followed by your call sign
b) Listen for 2 minutes before calling CQ
c) Send the letter "V" in Morse code several times and listen for a response or say "test" several
times and listen for a response
d) Send "QSY" on CW or if using phone, announce "the frequency is in use", then give your call
and listen for a response

***Updated by: 2/M LINO A. ARRIESGADO


Date: OCT. 10, 2018
81
6. There are no signals or static heard in the receiver of the SSB transceiver, although a loud noise is heard
if the volume control is advanced all the way up, what is the possible cause?
a) The antenna or feedline has been damaged.
b) The antenna switch has been transferred to the test position.
c) A defective receiver.
d) Any of these.

7. The urgent priority should be used for __________.


a) Messages concerning the Safety of Life At Sea (SOLAS).
b) Messages detailing important navigational warnings.
c) Messages containing information concerning the safety of a mobile unit or person.
d) Messages concerning On-scene communications.

8. Which of the following situations would normally use the Urgency priority?
a) Scenarios concerning the Safety of navigation or important meteorological warnings.
b) A crewmember over the side.
c) A serious medical situation involving a crewmember.
d) Both a and b

9. Which action is the most appropriate action for a GMDSS radio Operator to take in a distress situation
where immediate help is needed, but the vessel is not sinking nor needs to be abandoned?
a) Switch off EPIRB and SART manually.
b) Transmit distress call by HF/MF/VHF DSC or Inmarsat.
c) Notify the RCC (Rescue Coordination Center) through VHF FM on channel 13.
d) Transmit distress call by activating the radiotelegraph automatic alarm signal.

10. What does a SART signal sound or look like?


a) It transmits "SOS" and the vessel's name and position in slow speed Morse Code.
b) It will appear on a radar unit's PPI as a line of dots radiating outward with the innermost dot
indicating the SART's position.
c) It will appear on a radar unit's PPI as a line of dots radiating outward with the outermost dot
indicating the SART's position.
d) None of the above.

11. GMDSS vessels equipped for A2, A3 or A4 must maintain a continuous DSC watch on 2187.5 kHz.
a) Only in areas beyond Inmarsat coverage.
b) Only outside of areas covered by VHF-DSC.
c) When directed to do so by a cognizant rescue authority.
d) At all times when underway.

***Updated by: 2/M LINO A. ARRIESGADO


Date: OCT. 10, 2018
82
12. Which of the following situations would normally use the urgency priority?
a) Leaking oil from a minor tank fracture.
b) Treatment of crewmember breaking a leg in a cargo hold.
c) A fire in the generator flat/spaces.
d) Answers a) and b) are both possible.

13. Which of the following situations would normally use the Urgency priority?
a) A crewmember over the side.
b) A serious medical situation involving a crewmember.
c) A cargo shift or weather situation considered to be of greater hazard than would justify a
SAFETY priority designation.
d) Answers b) and c).

14. What is the fundamental concept of the GMDSS?


a) It is intended to automate and improve emergency communications in the maritime industry.
b) It is intended to automate and improve existing digital selective calling procedures and
techniques.
c) It is intended to provide more effective but lower cost commercial communications.
d) It is intended to provide compulsory vessels with a collision avoidance system when they are
operating in waters that are also occupied by non-compulsory vessels.

15. As in the event of partial satellite equipment breakdown, On Scene communications are usually initiated
using what equipment?
a) EPIRB on 121.5 MHz
b) SART on 9 GHz
c) VHF on Ch-16
d) VHF on Ch-70

16. If a vessel is engaged in local trade and at no point in its voyage travels outside the range of a VHF
shore station with continuous DSC alerting then the vessel is operating in what area?
a) Sea area A1
b) Coastal and international zones
c) Inland and coastal waters
d) Sea areas A1 and A2

17. In partial breakdown, GMDSS-equipped ships will be required to perform which of the following
communications functions?
a) Distress alerting, MSI, SAR and on-scene communications & receipt of satellite alerts from other vessels.
b) SAR and on-scene communications, Bridge-to-Bridge and general radio communications, MSI and relay
of satellite alerts from other vessels.
c) Bridge-to-Bridge and general radio communications, RDF of EPIRB homing signals, Distress alerting
and MSI.
d) Transmit distress alerts, SAR and on-scene communications, MSI, Bridge-to-Bridge and general radio
communications.
***Updated by: 2/M LINO A. ARRIESGADO
Date: OCT. 10, 2018
83
18. Which of the frequency is allocated as the GMDSS Digital Selective Calling Frequency?
a) 157.525 KHz
b) 2182 KHz
c) 156.300 KHz
d) 156.525 MHz

19. In the event of breakdown, the following are required GMDSS functions for vessels?
a) Transmit and receive locating signals, general communications and SAR communications.
b) Transmit and receive general communications, transmit Distress Alerts by at least one means,
MSI.
c) Transmit and receive locating signals, send MSI to other ships via EGC, Bridge-to-Bridge
communications.
d) Transmit and receive SAR communications, transmit Distress Alerts by at least one means,
Bridge-to-Bridge communications.

20. In the event of partial breakdown of communication facilities. What action(s) should be taken when
abandoning ship?
a) Send a VHF-DSC Distress alert on Ch-16 before going to the boats.
b) Activate the EPIRB and leave it secured to the mounting bracket.
c) Take EPIRB, SART and SCT units to the survival craft and use as circumstances dictate.
d) Once in the survival craft – activate the EPIRB and send a VHF-DSC Distress alert on
Ch-16

21. In case of terrestrial communication breakdown, which of the following satellite systems is of particular
& dedicated importance to search and rescue missions under GMDSS?
a) COSPAS/SARSAT
b) Inmarsat
c) GPS
d) Iridium

22. In case of equipment breakdown, which piece of required GMDSS equipment is the primary source of
transmitting locating signals?
a) Radio Direction Finder (RDF).
b) Survival Craft Transceiver.
c) An EPIRB transmitting on 406 MHz.
d) A SART transmitting on 406 MHz

23. The EPIRB is required to have a sufficient battery capacity to operate continues signal for what period
of time?
a) 12 Hours
b) 24 Hours
c) 36 Hours
d) 48 Hours

***Updated by: 2/M LINO A. ARRIESGADO


Date: OCT. 10, 2018
84
24. When in standby mode, the SART battery should last up to…
a) 8 hours
b) 24 hours
c) 48 hours
d) 96 hours

25. What is the term which refers to the supply of electrical energy required to supply radio
installation on every ship for the purpose of conducting distress and safety
radiocommunications in the event of failure of the ship’s main and emergency sources of
electrical power?
a) 110 volts primary supply
b) 220 volts primary supply
c) Reserve source of energy (or battery)
d) Secondary power supply

26. The SART is required to have a sufficient battery capacity to operate in the stand-by mode for what
period of time?
a) Eight hours
b) Forty eight hours
c) Four days
d) Three days

27. As a secondary method of alerting, what names are used to prevent the reception of unwanted broadcast
s by vessel’s utilizing the NAVTEX system?
a) Select all stations
b) Select all types of messages
c) Contact the NAVTEX coordinator
d) Programming the receiver to reject unwanted broadcasts

28. In the event of equipment breakdown, how long should the battery of the SART last when in Transpond
mode?
a) 4 hours
b) 6 hours
c) 8 hours
d) 10 hours

***Updated by: 2/M LINO A. ARRIESGADO


Date: OCT. 10, 2018
85
29. In partial breakdown of equipment, to maximize the effectiveness of VHF communications is by…
a) Appropriate setting of the transmitter power, selecting an appropriate channel & adjustment of
squelch for maximum receiver sensitivity.
b) The adjustment of squelch for maximum receiver sensitivity, setting transmitter power to 1W &
selecting an appropriate channel.
c) Selecting an appropriate channel, adjustment of squelch for minimum receiver sensitivity &
setting transmitter power to 1W.
d) Selecting an appropriate channel, adjustment of squelch for minimum receiver sensitivity, setting
transmitter power to 25W.

30. As a secondary method of alerting, SART is to be used for locating. What signals will be displayed in
3cm x-band radar if the distance is in between 0-0.5 nm?
a) Range of search craft at approximately 1 nm dots change to wide arcs.
b) SART showing 12 dot blip code bearing approximately 060 degrees.
c) SART showing more than 12 dot blips code
d) Wide arcs change to concentric circles as the SART is closed.

31. In partial breakdown, GMDSS-equipped ships will be required to perform which of the following
communications functions?
a) Distress alerting, MSI, SAR and on-scene communications & receipt of satellite alerts from other
vessels.
b) SAR and on-scene communications, Bridge-to-Bridge and general radio communications, MSI
and relay of satellite alerts from other vessels.
c) Bridge-to-Bridge and general radio communications, RDF of EPIRB homing signals, Distress
alerting and MSI.
d) Transmit distress alerts, SAR and on-scene communications, MSI, Bridge-to-Bridge and general
radio communications.

32. In the event of breakdown, the following are required GMDSS functions for vessels?
a) Transmit and receive locating signals, general communications and SAR communications.
b) Transmit and receive general communications, transmit Distress Alerts by at least one means,
MSI.
c) Transmit and receive locating signals, send MSI to other ships via EGC, Bridge-to-Bridge
communications.
d) Transmit and receive SAR communications, transmit Distress Alerts by at least one means,
Bridge-to-Bridge communications.

33. Under GMDSS, a compulsory VHF-DSC radiotelephone installation must be tested at what minimum
intervals at sea?
a) Annually, by a representative of the FCC.
b) Daily
c) At the annual SOLAS inspection.
d) Monthly

***Updated by: 2/M LINO A. ARRIESGADO


Date: OCT. 10, 2018
86
34. At sea, all required equipment (other than Survival Craft Equipment) must be proven operational by:?
a) Daily testing or daily successful use of the equipment.
b) Testing at least every 48 hours.
c) Weekly testing of all S.C.E. and other compulsory equipment.
d) Daily testing of the S.C.E. and weekly tests of the other equipment.

35. What is the requirement for emergency and reserve power in GMDSS radio installations?
a) Compulsory ships must have emergency and reserve power sources for radio
communications.
b) An emergency power source for radio communications is not required if a vessel has proper
reserve power (batteries).
c) A reserve power source is not required for radio communications.
d) Only one of the above is required if a vessel is equipped with a second 406 EPIRB as a backup
means of sending a Distress alert.

36. Which statement is NOT true regarding the requirements of survival craft portable two-way VHF
radiotelephone equipment?
a) Operation on Ch-16
b) Antenna must be permanently-affixed.
c) Simplex (single frequency) voice communications only.
d) Effective radiated power should be a minimum of 2.0 Watts.

37. What are the main points of sending distress message following distress signals?
a) Identification of the vessel in distress
b) Weather condition in the immediate area of the ship
c) Number of crew and passengers
d) Navigational hazards

38. According to the standard marine navigational vocabulary, the message marker which indicates that the
following message informs other traffic participants about danger is?
a) Alerting
b) Messaging
c) Transmitting
d) Warning

39. According to the standard marine navigational vocabulary, the message marker which indicates that the
following message informs other traffic participants about danger is?
a) Information
b) Warning
c) Advice
d) Intention

***Updated by: 2/M LINO A. ARRIESGADO


Date: OCT. 10, 2018
87
40. In Seaspeak, the suitable message marker to preface the message. The ship ahead of you is not under
command? is:
a) Information
b) Advice
c) Warning
d) Instruction

41. What is the frequency of a medium size vessel’s whistle have?


a) 70 and 200 Hz
b) 80 and 200 Hz
c) 90 and 200 Hz
d) 100 and 200 Hz

42. The fundamental frequency of the whistle for a vessel 200 meters or more in length must be between;
a) 70 and 200 Hz
b) 80 and 200 Hz
c) 90 and 200 Hz
d) 100 and 200 Hz

43. In case of full breakdown for a vessels of less than 75 meters, what is the whiste frequency which will
give a relatively shrill tone?
a) 150 and 700 Hz
b) 200 and 700 Hz
c) 250 and 700 Hz
d) 300 and 700 Hz

44. For a intermediate size of vessel, what will be the whistle frequency required?
a) 110-350 Hz
b) 110-450 Hz
c) 130-350 Hz
d) 130-450 Hz

45. Which of the following statements concerning testing and maintenance of SARTs is true?
a) Testing a SART should be done in a consistent manner & location to ensure a baseline history of
proper results.
b) Testing of the SART should never be done in port to prevent interference to other vessel’s
radars.
c) A SART's battery must be replaced within ninety (90) days after the expiration date imprinted on
the unit.
d) An at-sea GMDSS maintainer is not able to test a SART because it is hermetically sealed.

***Updated by: 2/M LINO A. ARRIESGADO


Date: OCT. 10, 2018
88
46. Why should functional testing of a SART be minimized?
a) Potential interference with safe navigation, notifying other vessels of an actual Distress and
minimize power consumption.
b) Minimize power consumption of the battery and only test at sea to reduce potential interference
or confusion.
c) Possibility of misinterpretation by other vessels as a Distress situation and only test in port to
prevent potential interference with safe navigation or at-sea vessels.
d) Potential interference with safe navigation, possible misinterpretation of an actual Distress,
minimizes draining the battery.

47. Why is it important to limit the duration of testing a SART?


a) Excessive testing causes "burn in" on the vessel's radar display.
b) Testing in port or even at sea may cause interference to other radars or a test signal may be
misinterpreted as a genuine Distress situation.
c) To prevent overheating, a SART requires sufficient ventilation that is significantly reduced when
the SART is being tested.
d) If another SART is testing at the same time, the two signals will cause damage to the unit that
transmitted them.

48. What statement is true regarding tests and maintenance that could be provided for the SART?
a) Full verification within manufacturer's specifications by the on-board maintainer would be a
requirement for all vessels in the A3 & A4 sea areas using measuring equipment to generate 9
GHz signals.
b) Battery should be replaced within the 90 day grace period following the manufacturer's
expiration date shown on the SART and the SART should only be tested at-sea to reduce
interference to other vessels.
c) Extreme care should be exercised because testing of the SART may be received by other
vessels, may be interpreted as a Distress condition, or it may interfere with other vessels' safe
navigation.
d) Battery should be replaced with a new one before the manufacturer's expiration date shown on
the SART and the SART should only be tested in port to reduce interference to other vessels.

49. Why should functional testing of a SART be minimized?


a) Potential interference with safe navigation, notifying other vessels of an actual Distress and
minimize power consumption.
b) Minimize power consumption of the battery and only test at sea to reduce potential
interference or confusion.
c) Possibility of misinterpretation by other vessels as a Distress situation and only test in port to
prevent potential interference with safe navigation or at-sea vessels.
d) Potential interference with safe navigation, possible misinterpretation of an actual Distress,
minimizes draining the battery.

***Updated by: 2/M LINO A. ARRIESGADO


Date: OCT. 10, 2018
89
50. Which is NOT a valid maintenance and testing function for a SART?
a) Operational test with several vessels to determine effective transmitting range.
b) Inspection of container for apparent damage.
c) Inspect battery expiration date and the lanyard condition.
d) Brief operational test utilizing own ship's radar.

51. Which statement is NOT true regarding the requirements of survival craft portable two-way VHF
radiotelephone equipment?
a) Watertight to a depth of 1 meter for 5 minutes.
b) Operates simplex on Ch-70 and at least one other channel.
c) Effective radiated power should be a minimum of 0.25 Watts.
d) The antenna is fixed and non-removable.

52. In Morse Code signaling by Hand Flag or Arms, following are the transmitting signals to a receiving
station except for;
a) To call up a station
b) A receiving station – in answer
c) Completing the signal
d) Sending the distress message

53. You have a Manoverboard situation, What would be the priority of the DSC call you are going to send?
a) Safety
b) Urgency
c) Distress
d) Routine

54. In visual signaling, what procedural signals to call unknown station?


a) The general call AA AA AA is made by transmitting station.
b) The directional light of an Aldis lamp
c) The Identity signal is to be made by receiving station
d) The receiving station should acknowledge

55. In partial breakdown of equipment, to maximize the effectiveness of VHF communications is by…
a) Appropriate setting of the transmitter power, selecting an appropriate channel & adjustment of
squelch for maximum receiver sensitivity.
b) The adjustment of squelch for maximum receiver sensitivity, setting transmitter power to 1W &
selecting an appropriate channel.
c) Selecting an appropriate channel, adjustment of squelch for minimum receiver sensitivity &
setting transmitter power to 1W.
d) Selecting an appropriate channel, adjustment of squelch for minimum receiver sensitivity, setting
transmitter power to 25W.

***Updated by: 2/M LINO A. ARRIESGADO


Date: OCT. 10, 2018
90
56. To exhibit a distress signal without using the GMDSS sub-system (equipment), which of the following
should be executed as to get attention of nearby vessels?
a) Parachute red flare
b) The use of EPIRB
c) The use of Navtex receiver
d) None of the above

57. While at anchor, which of the following signals are more visible for other vessel that your vessel is
under pirate attack?
a) Flames on a vessel
b) Gun fired at intervals of 1 minute
c) Orange smoke
d) Wave arms

58. As a secondary method of alerting, SART is to be used for locating. What signals will be displayed in
3cm x-band radar if the distance is in between 0-0.5 nm?
a) Range of search craft at approximately 1 nm dots change to wide arcs.
b) SART showing 12 dot blip code bearing approximately 060 degrees.
c) SART showing more than 12 dot blips code
d) Wide arcs change to concentric circles as the SART is closed.

59. Transmission of a Distress alert by a station on behalf of another vessel actually in Distress should not
occur:
a) When communications between the Distress vessel and a Coast station are already in progress.
b) When the mobile unit actually in Distress is not itself in a position to transmit the Distress alert.
c) When the Master or responsible person on the mobile unit not in Distress so decides.
d) When the responsible person at the Coast Station determines further help is necessary.

60. You are in voice communication on Ch-16 with a vessel in Distress that advises you they are unable to
contact a Coast Station. What action would you take?
a) Send a DSC Distress Relay transmission on Ch-16.
b) Attempt to contact a Coast Station using voice on Ch-16 with a Mayday Relay.
c) Make an all ships call with Urgency priority.
d) Call the Coast Station on Ch-70 with Distress priority giving the other vessel’s position.

61. The EPIRB on the bridge wing is observed with the strobe light flashing and the control switch in the
“OFF” position. What action should be taken?
a) Place the control switch in the "OFF” position.
b) No action is needed – the strobe light indicates the automatic monthly self test is in progress.
c) Wait for the USCG or NOAA to confirm that the unit is actually transmitting.
d) Assume the unit is transmitting and follow the recommended procedures to properly cancel a
false distress alert being broadcast from an EPIRB.

***Updated by: 2/M LINO A. ARRIESGADO


Date: OCT. 10, 2018
91
62. Which of the following steps should be taken, if possible, when the vessel must be abandoned because
of a Distress situation?
a) Alert the U.S. Coast Guard by using the survival craft's portable Inmarsat unit.
b) Program the SART and EPIRB to transmit the vessel's location and situation.
c) No additional steps are needed as the SART and EPIRB will both automatically float free and
operate properly.
d) Secure the EPIRB to the survival craft and mount the SART in a position to maximize its
elevation.

63. If your vessel has suffered a casualty severe enough to disable both ship’s power and the GMDSS
console RSE you should:
a) Activate the EPIRB and/or use the SCT to make a “Mayday” call on Ch-16.
b) Use UHF Transceivers to contact other vessels.
c) Activate the EPIRB and/or use the SCT to make a “Mayday” call on Ch-70.
d) Make a “Mayday” call on Ch-70 and Ch-06 using the Survival Craft Transceiver.

64. Which of the following would most likely not prevent a SART's signal from being detected?
a) The rescue personnel were monitoring the 3-CM radar and the SART was mounted improperly
in the lifeboat.
b) The SART was mounted improperly in the survival craft and rescue personnel were monitoring
the 10-CM radar.
c) The rescue personnel were monitoring the 10-CM radar and the SART was properly mounted in
the lifeboat.
d) The SART was properly mounted in the lifeboat and rescue personnel were monitoring the 3-CM
radar.

65. Which statement is false regarding the maintenance of GMDSS equipment at sea?
a) The GMDSS maintainer may not be the person designated to have primary responsibility for
radio-communications during Distress incidents even if licensed as an operator.
b) Ships must carry at least one person who qualifies as a GMDSS maintainer for the maintenance
and repair of equipment if the at-sea maintenance option is selected.
c) All at-sea maintenance and repairs must be performed by, or under the supervision of a person
holding a GMDSS Maintainer license.
d) The GMDSS maintainer may be the person responsible for ensuring that the watches are
properly maintained and that the proper guard channels and the vessel's position are entered into
the DSC equipment.

66. What are the conditions, under GMDSS, whereby a ship is NOT allowed to depart from any port?
a) The vessel has a temporary waiver of its radio license and Safety Certificate
b) The vessel is carrying more than the required number of qualified GMDSS radio operators.
c) The vessel is not capable of performing all required Distress and Safety functions.
d) The vessel is not carrying a GMDSS radio maintainer, but has provided for shoreside
maintenance plus duplication of equipment if required.

***Updated by: 2/M LINO A. ARRIESGADO


Date: OCT. 10, 2018
92
67. Which statement is false regarding the radio operator requirements for a GMDSS-equipped ship station?
a) Maintaining a record of all incidents connected with the radio-communications service that
appear to be of importance to Safety of life at sea is not required.
b) One of the qualified GMDSS radio operators must be designated to have primary responsibility
for radio-communications during Distress incidents.
c) A qualified GMDSS radio operator, and a qualified backup, must be designated to perform
Distress, Urgency and Safety communications
d) While at sea, all adjustments or radio installations, servicing or maintenance of such installations
that may affect the proper operation of the GMDSS station must be performed by, or under the
supervision of, a qualified GMDSS radio maintainer.

68. Which of the Two-Letter signals are the most commonly used as “I require immediate assistance”?
a) AL
b) AN
c) CD
d) FR

69. In the use of visual signaling, which Morse Code regular procedural signal are to be applied using two
letters code for waiting signal?
a) AA
b) AB
c) AR
d) AS

70. The two-letter signal which means “I am on fire” is:


a) AC
b) YU
c) IT
d) CD

71. The two-letter signal which means “ I am in distress and require immediate assistance ” is:
a) CN
b) CD
c) SS
d) NC

72. What is the proper procedure to be followed upon receipt of a Distress alert transmitted by use of Digital
Selective Calling techniques?
a) Set watch on the DSC alerting frequency in the band of frequencies the alert was received.
b) Set a continuous watch on VHF-FM Channel 13, 16 and DSC on Channel 70.
c) Ship stations equipped with narrow-band direct-printing equipment should respond to the
Distress alert as soon as practicable by this means.
d) Set watch on the radiotelephone Distress and Safety frequency associated with the Distress and
Safety calling frequency on which the Distress alert was received.
***Updated by: 2/M LINO A. ARRIESGADO
Date: OCT. 10, 2018
93
73. You are in voice communication on Ch-16 with a vessel in Distress that advises you they are unable to
contact a Coast Station. What action would you take?
a) Send a DSC Distress Relay transmission on Ch-16.
b) Attempt to contact a Coast Station using voice on Ch-16 with a Mayday Relay.
c) Make an all ships call with Urgency priority.
d) Call the Coast Station on Ch-70 with Distress priority giving the other vessel’s position.

74. Which of the following steps should be taken, if possible, when the vessel must be abandoned because
of a Distress situation?
a) Alert the U.S. Coast Guard by using the survival craft's portable Inmarsat unit.
b) Program the SART and EPIRB to transmit the vessel's location and situation.
c) No additional steps are needed as the SART and EPIRB will both automatically float free and
operate properly.
d) Secure the EPIRB to the survival craft and mount the SART in a position to maximize its
elevation.

75. If your vessel has suffered a casualty severe enough to disable both ship’s power and the GMDSS
console RSE you should:
a) Activate the EPIRB and/or use the SCT to make a “Mayday” call on Ch-16.
b) Use UHF Transceivers to contact other vessels.
c) Activate the EPIRB and/or use the SCT to make a “Mayday” call on Ch-70.
d) Make a “Mayday” call on Ch-70 and Ch-06 using the Survival Craft Transceiver.

***Updated by: 2/M LINO A. ARRIESGADO


Date: OCT. 10, 2018
94
GOC FOR GMDSS REVIEWER
COMPETENCE 2 Provide Radio Services in Emergencies (GOC)
KUP4 – PREVENTIVE MEASURES FOR SAFETY OF SHIP AND PERSONNEL IN
CONNECTION WITH HAZARDS RELATED TO RADIO EQUIPMENT, INCLUDING
ELECTRICAL AND NON-IONIZING HAZARDS,

1) Batteries should not be topped-up:


a) Whilst on charge
b) During idle
c) During discharge
d) Whilst in use

2. It is recommended that eye protection, gloves, etc. be worn when a person is carrying out maintenance
on batteries.
a) Coverall and Jacket
b) Skin protection and face
c) Colored eyed protection,gloves and apron
d) Eye protection, gloves, and “APRON”

3. The electrolyte in battery cells contains ______, it is sufficiently concentrated, particularly just after
charging to damages eyes, skin or clothes if split or splashed, immediate and prolonged application of
running water is recommended to minimize its effect:
a) Alcohol denature
b) Acetone
c) Sulfuric acid
d) Hydrochloride

4. There are two hazards associated with lead acid batteries that ship operators should be aware of:
a) Risk of explosion and risk of chemical burns
b) Risk of fire and explosion
c) Risk of suffocation and poison
d) Risk of collision and breakdown

5. A-B-C first aid procedure in dealing electrical shock stands for?


a) Air, Breath, Circulatory
b) Air, Breaking, Circulating
c) Airway, Bathing, Circulation
d) Airway, Breathing, Circulation

***Updated by: 2/M LINO A. ARRIESGADO


Date: OCT. 10, 2018
95
6. During the chemical process in battery charging the highly explosive mixture produced during the
chemical process may form highly explosive mixture:
a) When exposed to naked flame, fire, a lighted cigarette or spark.
b) Mixture to carbon oxide
c) Exposed to heat, smoke and chemical reaction
d) Flammable mixture

7. Extreme care should be made when using _____tool:


a) Metal
b) Plastic
c) Wood
d) Iron plate

8. If vessel is at anchor or moored, the ships station may communicate station or limited stations maybe
permitted:
a) Is permitted providing the minimum practicable transmitting power is used.
b) With high power setting
c) Is allowed only at anchorage
d) At all times

9. As a result of the chemical process occurring within the cells of battery during charging, what gas is
produced?
a) Oxygen
b) Nitrogen
c) Hydrogen
d) acetylene

10. Ship station license authorize the use of radio equipment aboard vessels:
a) Specifies the call sign of the station
b) Only at sea/inland waters.
c) Specifies the frequencies to be used.
d) At all times

11. Due to the risk of explosion, radio transmissions when a vessel during loading fuel, or when discharging
or loading dangerous or any flammable cargo.
a) Must be done
b) Must be perform
c) Must not be made
d) To be done at all times

***Updated by: 2/M LINO A. ARRIESGADO


Date: OCT. 10, 2018
96
12. When topping up the battery cells of Lead Acid Battery, _______has to be used to avoid introducing any
extraneous chemicals into the electrolyte, which could block the chemistry of the charging/discharging
process.
a) Battery solution
b) Electrolyte
c) Dilute electrolyte
d) Distilled or demineralised water

13. What is the primary functions of seamen’s insurance law under article III :
a) To provide radio medical consultation by seamen’s insurance radio medical center
b) To provide medical check- up to seafarer before joining vessel
c) To provide medical check-up to all persons
d) To provide medical assistance free of charge

14. What year seamen’s insurance law started:


a) Feb 1, 1992
b) Feb 1, 1999
c) April 1, 1963
d) Feb 1, 1995

15. Enumerate the basic vital signs of the patient if he/she is still alive:
a) Pulse
b) Breathing
c) Body temperature/Blood pressure
d) All of the above

16. Medical kits carried by ships are classified into how many types:
a) 6
b) 3
c) 4
d) 5

17. In GMDSS, Radio Medical Services is stipulated under column-2 of table A-IV/2 of STCW code is
listed under what sequence no:?
a) .1
b) .2
c) .3
d) .4

***Updated by: 2/M LINO A. ARRIESGADO


Date: OCT. 10, 2018
97
18. In sending Radio Medical advice how many essential medical data are required by the Radio Medical
Center hospital to diagnose the patient’s health condition:
a) 5
b) 8
c) 9
d) 10

19. What NOT to do when giving first aid:


a) Do not give first aid if you have doubts about your ability
b) Do not enter an enclosed space unless you are sure it is safe
c) Do not remove the person without checking body condition
d) All of the above

20. Basic life support restores two vital functions. It uses an ABC sequence of actions to ensure an open:
a) AIRWAY
b) BREATHING
c) BLOOD CIRCULATION
d) ALL OF THE ABOVE

21. The purpose of CPR is to restore the patient’s:


a) BREATHING
b) BLOOD CIRCULATION
c) To open airway
d) To lower cholesterol level

22. In administering CPR one rescuer should apply chest compression per minute:
a) 50
b) 60
c) 100
d) 30

23. Relative to question no.32, two rescuer should apply chest compression per minute:
a) 50
b) 60
c) 100
d) 30

24. Relative to question no. 32,how many rescue breath must be applied to the patient to restore his
breathing:
a) 3
b) 4
c) 2– every 15 chest compression
d) 5

***Updated by: 2/M LINO A. ARRIESGADO


Date: OCT. 10, 2018
98
25. Relative to question no. 33, how many rescue breath must be applied to the patient to restore his
breathing:
a) 1- every 5 chest compression
b) 4
c) 3
d) 2

26. What part of an injured person the rescuer would take a pulse:
a) CAROTID (NECK)
b) STOMACH
c) HEAD
d) BUTTOCK

27. CHOKING is the result of an obstruction in what part of the body:


a) NOSE
b) EAR
c) LARYNX/TRACHEA
d) ALL OF THE ABOVE

28. BLEEDING is the result of damage to what part of the body:


a) WOUND
b) DAMAGE TO BLOOD VESSEL
c) HEAD INJURY
d) PEPTIC ULCER

29. What are the most commonly used important Two-letter signals which specify “I have a doctor
onboard”?
a) AL
b) AN
c) CB
d) FR

30. In the EMS system, how many “key-points” communications are required for essential and effective
EMS communication:
a) 3
b) 10
c) 7
d) 5

***Updated by: 2/M LINO A. ARRIESGADO


Date: OCT. 10, 2018
99
31. The radio communication equipment may operate either from the ship’s DC or AC mains supply (often
stepped down to 24 V DC), or from 24 V DC supplied by a bank of batteries. As a preventive measures,
what do you call for the system that ensures batteries will automatically take over when mains supply
fails?
a) Boost charging system
b) Float charging system
c) Dummy charging system
d) Fast charging system

32. In the event of breakdown, reserve sources of energy should be adequate for at least how hours for the
conduction of operation, complying with SOLAS Ch. 11-1/42 or 43 and Ch. IV/13.2.1 and 13.2.2, as
appropriate?
a) One hour or six hours
b) Two hours or seven hours
c) Three hours or eight hours
d) Four hours or nine hours

33. The measurement for battery capacity, at a temperature of 20°C is ampere-hour (AH).
That means, that theoretically a battery of cells, in a good condition, rated at 140 AH can deliver 10
amperes for how many hours of operations?
a) 5 hours
b) 10 hours
c) 15 hours
d) 20 hours

34. In case of breakdown, the batteries should be charged at least in a minimum charging time with the
value of the average of the charging current should measure 10% of value of the battery’s capacity?
a) 5 hours
b) 10 hours
c) 15 hours
d) 20 hours

35. What are the voltage requirement mainly used for reserve source of energy in case of power failure from the main
supply?
a) 12 volts AC
b) 12 volts DC
c) 24 volts AC
d) 24 volts DC

***Updated by: 2/M LINO A. ARRIESGADO


Date: OCT. 10, 2018
100
36. Which of the primary battery has a characteristic that the cells consist of a zinc cover as the negative
electrode?
a) Lead Acid Batteries
b) Lithium Batteries
c) Nickle-Cadmium Batteries
d) Zinc Carbon Cell Batteries

37. What type of batteries most commonly used on board ships as source of back-up power?
a) Lead Battery
b) Lithium Battery
c) Nickel-Cadium (NiCd) Battery
d) Nickel-iron (NIFe) Battery

38. The type of batteries that is damaged by either over-charging or under charging is the…?
a) Gel type battery
b) Lead acid battery
c) Lithium battery
d) Nickel-Cadium battery

39. Marine lead batteries have an approximate lifetime of…


a) 2 years
b) 3 years
c) 4 years
d) 5 years

40. What is the charge of Proton?


a) Negative
b) Positive
c) Neutral
d) No charge

41. Big 3 in electricity?


a) Current
b) Amper
c) Ohms
d) Voltage

***Updated by: 2/M LINO A. ARRIESGADO


Date: OCT. 10, 2018
101
42. As a preventive measures, the following rules to be obeyed during operation, control and emergency
lighting service of the storage battery, except for...?
a) Do not allow the battery to stand idle for long time this may cause inactivation of the battery cells.
b) Do not charge the battery with very high rate of current because high rate of charging causes high
temperature rise and excessive gassing resulting in heavy loss of water and sometime overflowing of
electrolyte from the battery cells.
c) After every complete discharge, the battery should be immediately charged before returning it to its
regular floating service. Otherwise there may be a chance of deposition of sulphate film on the
plates.
d) The specific gravity of the electrolyte may be permanently increased due to ageing effects.

43. The defining characteristics of various types of cells are the cell (battery) voltage and the battery
capacity. The cell voltage is open-circuit potential difference between the electrodes (also called
electromotive force), Which force of cell voltage is open-circuit between electrodes?
a) Electromagnetic force
b) Electromotive force
c) Electrostatic force
d) Electrochemical force

44. In the event of failure of the main and emergency sources of electrical power –
what is the term for the source required to supply the GMDSS console with power for
conducting Distress and other radio-communications?
a) Emergency power
b) Reserve source of energy
c) Ship's emergency diesel generator
d) Ship's standby generator

45. What is the requirement for emergency and reserve power in GMDSS radio installations?
a) Compulsory ships must have emergency and reserve power sources for radio communications.
b) An emergency power source for radio communications is not required if a vessel has proper
reserve power (batteries).
c) A reserve power source is not required for radio communications.
d) Only one of the above is required if a vessel is equipped with a second 406 EPIRB as a backup
means of sending a Distress alert.

46. In visual signaling, what method of communication uses two hand flags to indicate letters of the
alphabet?
a) Flag signaling
b) Flashing light signaling
c) Semaphore or hand flags or arms signaling
d) Sound signaling

***Updated by: 2/M LINO A. ARRIESGADO


Date: OCT. 10, 2018
102
47. In flag signaling, this uses to describe a signal which is hoisted to approximately half the extend of the
halyard.
a) At the dip
b) Close-up
c) Signal Letters
d) Tackline

48. In flag signaling, a hoist or signal is said to be close up when it is hoisted…


a) about half of the full extent of the halyards
b) about third of the full extent of the halyards
c) about fourth of the full extent of the halyards
d) To the full extent of the halyards

49. What system that is based in computer who save or keep a record of all crewmembers who seek medical
advice/assistance?
a) RMSS
b) SEMPOS
c) GMDSS
d) SOLAS

50. Which of the following statements concerning testing and maintenance of SARTs is true?
a) Testing a SART should be done in a consistent manner & location to ensure a baseline history of proper
results.
b) Testing of the SART should never be done in port to prevent interference to other vessel’s radars.
c) A SART's battery must be replaced within ninety (90) days after the expiration date imprinted on the unit.
d) An at-sea GMDSS maintainer is not able to test a SART because it is hermetically sealed.

51. Why is it important to limit the duration of testing a SART?


a) Excessive testing causes "burn in" on the vessel's radar display.
b) Testing in port or even at sea may cause interference to other radars or a test signal may be
misinterpreted as a genuine Distress situation.
c) To prevent overheating, a SART requires sufficient ventilation that is significantly reduced when
the SART is being tested.
d) If another SART is testing at the same time, the two signals will cause damage to the unit that
transmitted them.

52. What statement is true regarding tests and maintenance that could be provided for the SART?
a) Full verification within manufacturer's specifications by the on-board maintainer would be a requirement
for all vessels in the A3 & A4 sea areas using measuring equipment to generate 9 GHz signals.
b) Battery should be replaced within the 90 day grace period following the manufacturer's expiration date
shown on the SART and the SART should only be tested at-sea to reduce interference to other vessels.
c) Extreme care should be exercised because testing of the SART may be received by other vessels, may be
interpreted as a Distress condition, or it may interfere with other vessels' safe navigation.
d) Battery should be replaced with a new one before the manufacturer's expiration date shown on the SART
and the SART should only be tested in port to reduce interference to other vessels.
***Updated by: 2/M LINO A. ARRIESGADO
Date: OCT. 10, 2018
103
53. Why should functional testing of a SART be minimized?
a. Potential interference with safe navigation, notifying other vessels of an actual Distress and
minimize power consumption.
b. Minimize power consumption of the battery and only test at sea to reduce potential interference
or confusion.
c. Possibility of misinterpretation by other vessels as a Distress situation and only test in port to
prevent potential interference with safe navigation or at-sea vessels.
d. Potential interference with safe navigation, possible misinterpretation of an actual Distress,
minimizes draining the battery.

54. Which of the following is normally part of 406 MHz satellite EPIRBs?
a) A strobe light, automatic float-free bracket, 1-watt 406-MHz alert beacon.
b) A 5-watt 406-MHz alert beacon, Automatic Hydrostatic Release (ARM), strobe light.
c) Automatic float-free bracket, 5-watt 121.5 MHz homing beacon, strobe light.
d) Automatic Hydrostatic Release (ARM), 1-watt 121.5 MHz alerting beacon, strobe light.

55. Which of the following control selections may result in limited receiving range?
a) Setting the squelch control to its minimum level.
b) The power switch is set to the "high" output position resulting in receiver overloading.
c) Setting the squelch control to its maximum level.
d) Setting the channel selection switch midway between channels 6 and 16.

56. Equipment for radiotelephony use in survival craft stations under GMDSS must have what
characteristic(s)?
a) Permanently-affixed antenna, watertight, power 1W or 25W.
b) Watertight, power a minimum of 1W, operation on CH-16, Ch-13 & Ch-70.
c) Operation on Ch-16, watertight, permanently-affixed antenna.
d) Operation on Ch-16, Ch-13 & Ch-70, power 1W, permanently-affixed antenna.

57. Which statement is NOT true regarding the requirements of survival craft portable two-way VHF
radiotelephone equipment?
a) Operation on Ch-16
b) Antenna must be permanently-affixed.
c) Simplex (single frequency) voice communications only.
d) Effective radiated power should be a minimum of 2.0 Watts.

58. Which statement is NOT true regarding the requirements of survival craft portable two-way VHF
radiotelephone equipment?
a) Watertight to a depth of 1 meter for 5 minutes.
b) Operates simplex on Ch-70 and at least one other channel.
c) Effective radiated power should be a minimum of 0.25 Watts.
d) The antenna is fixed and non-removable.

***Updated by: 2/M LINO A. ARRIESGADO


Date: OCT. 10, 2018
104
59. How is mutual interference on 518 kHz among NAVTEX stations avoided?
a) Transmissions scheduled on a time-sharing basis, power limited and station assignment codes are
geographically separated.
b) All stations transmit at the same time but stations are limited to daytime operation only to reduce
the radius of propagation.
c) Transmitter power is limited, station assignment codes are not shared by other NAVAREAS and
stations alternate between daytime and nighttime operations.
d) Station codes are not shared by other NAVAREAS, transmissions scheduled on a time-sharing
basis and power is limited.

60. While conducting routine communications using the wheelhouse VHF with a station 1 mile distant, your
recommended power setting would be:?
a) 25 watts after dark.
b) 25 watts during a clear sunny day.
c) 1 watt, day or night.
d) 1 watt using DSC at night.

61. Which of the following factors does not normally affect the range of VHF transmissions?
a) Salt water ingress into the antenna coaxial cable.
b) Power level setting.
c) Vessel antenna height.
d) Ionospheric refraction.

62. Much longer than normal VHF transmissions are typically caused by:?
a) Atmospheric ducting or tropospheric propagation.
b) Changing power from 1W to 25 W.
c) Skywave reflections from the D layer.
d) Ionospheric activity in layers F1/F2.

63. Describing VHF transmissions as "line of sight" does not mean:?


a) Vessel antenna height will not affect the radius of propagation.
b) The normal transmission range to a coast station is approximately is 10 NM.
c) Coast station antenna height has no effect on the radius of transmission.
d) VHF communications are effective only with nearby stations within visual range of the bridge.

64. The effectiveness of VHF communications is maximized by:?


a) Appropriate setting of the transmitter power, selecting an appropriate channel & adjustment of
squelch for maximum receiver sensitivity.
b) The adjustment of squelch for maximum receiver sensitivity, setting transmitter power to 1W &
selecting an appropriate channel.
c) Selecting an appropriate channel, adjustment of squelch for minimum receiver sensitivity &
setting transmitter power to 1W.
d) Selecting an appropriate channel, adjustment of squelch for minimum receiver sensitivity, setting
transmitter power to 25W.
***Updated by: 2/M LINO A. ARRIESGADO
Date: OCT. 10, 2018
105
65. The VHF-DSC self-test function (if available) usually performs the following:?
a) Transmits a very weak DSC signal which is picked up by the Ch-70 receiver.
b) Transmits a weak voice signal which is picked up on Ch-16.
c) Makes a diagnostic test of the system without actually transmitting a signal.
d) Does a check sum of the memory chips.

66. Which statement best describes frequency modulation?


a) Both the amplitude and frequency are changed by the modulating signal.
b) The frequency is changed by the information signal and the amplitude remains unchanged.
c) Frequency modulation is subject to interference by atmospheric noise.
d) High level mixing of the final amplifier signal and the information signal.

67. In Radiowave propagation, the distance traveled by a Radiowave from one wavetop to another is
called;?
a) Frequency
b) Wavelength
c) Magnetic field
d) Static duration

68. How can rescue personnel detect that a SART is transmitting in the immediate vicinity?
a) The SART’s blips on the PPI will begin showing series of dots and eventually become
concentric circles
b) The SART’s blips on the PPI will begin showing rapid dashes and eventually become
concentric circles
c) Both a and b
d) The SART’s blips on the PPI will begin arcing and eventually become concentric circles

69. How is mutual interference among NAVTEX stations avoided?


a) Transmitting at maximum power output
b) Transmit MSI once every 24 hours
c) Transmitter power is limited to that necessarily for coverage of assigned area, transmissions
by stations in each NAVAREA are arranged in a time – sharing basis
d) Transmit message type A, B and D only

70. What should an operator do to prevent interference?


a) Do not transmit
b) Adjust squelch
c) Monitor channel before transmitting and make transmission as brief as possible
d) Increase volume

***Updated by: 2/M LINO A. ARRIESGADO


Date: OCT. 10, 2018
106
71. When listening to the transmission of a very close station, what step must be observed to prevent the
strong signal from “smothering” the receiver?
a) Lower power output
b) Switch off speaker and use headset
c) Adjust volume control
d) Adjust squelch

72. Which of the following statements concerning a fixed RADIOTELEPHONE?


a) Messages detailing important navigational warnings
b) Fixed units operate in the routine mode with RF power level no greater than 1 watt
c) A serious medical situations involving a crewmember
d) Scenarios concerning the safety of navigation of important meteorological warnings.

73. What are the operation of emergency equipment & application of emergency procedures?
a) Distress signal
b) Routine signal
c) Safety Signal
d) Urgency Signal

74. In Morse Code signaling by Hand Flag or Arms, following are the transmitting signals to a receiving
station except for;?
a) To call up a station
b) A receiving station – in answer
c) Completing the signal
d) Sending the distress message

75. When two 12-volt batteries with an individual capacity of 100 Ah are coupled in parallel, the result will
be a battery with a total voltage and capacity of?
a) 12 volt and 200 ah respectively
b) 12 volt and 100 Ah respectively
c) 24 volt and 100 ah respectively
d) 24 volt and 200 ah respectively

76. A frequent maintenance is the basis for a reliable working condition of the battery, when working on
batteries, effective safety precautions must be taken except for;
a) Avoid over-discharging below 2.1 Volts for any cell
b) Ensure electrolyte level is maintained, but do not overfill, 1 cm above plates is adequate
c) Keep cells top clean and dry, check ventilation holes, tighten terminals and coat with
Vaseline
d) Charge the batteries for more than specified.

***Updated by: 2/M LINO A. ARRIESGADO


Date: OCT. 10, 2018
107
77. Following are the safety measures to be taken during handling storage battery in substation except
for…?
a) Do not bring a flame inside the room
b) Do not allow the battery to stand idle for long time
c) Do not generate any spark inside the room
d) Wear plash proof googles, rubber gloves, while workinh with the battery

78. The lead acid battery SPECIFIC GRAVITY should be tested by means of hydrometer at least:
a) Once a day
b) Once a week
c) once a month
d) once a year

79. The connection and condition of the reserve batteries should be checked:
a) Once a month
b) once a year
c) once every three months
d) once a week

80. What is the capacity of a 12V battery if it is supply a load of 10 amperes for 6 hours continuous
operation?
a) 38 Ah
b) 60 Ah
c) 72 Ah
d) 120 Ah

81. When two 12-volt batteries with an individual capacity of 100 Ampere-hours (Ah) are coupled in series,
the result will be a battery with a total voltage and capacity of…
a) 12-volt and 100 Ah respectively
b) 12-volt and 200 Ah respectively
c) 24-volt and 100 Ah respectively
d) 24-volt and 200 Ah respectively

82. The lead acid battery VOLTAGE should be tested at least:


a) Once a day
b) once a week
c) once a month
d) once a year

***Updated by: 2/M LINO A. ARRIESGADO


Date: OCT. 10, 2018
108
83. What are the types of electromagnetic radiation with enough energy to ionize biological materials.
a) Chemical reactive radiation
b) Radioactive radiation
c) Thermal radiation
d) X-radiation and gamma radiation

84. What are the examples of ionizing radiations?


a) Chemical reactive radiation
b) Radioactive radiation
c) Thermal radiation
d) X-radiation and gamma radiation

85. What are the Radiofrequency radiation that generate energy via water molecules results in rapid heating
through out an object?
a) Microwave radiation
b) Radioactive radiation
c) X-radiation and gamma radiation
d) Thermal radiation

86. What are the example of a non-telecommunication use of RF energy.


a) Microwave oven
b) Satellite parabolic
c) Whip antenna
d) Radar antenna

87. Which of the following systems is most likely to be subject to fading or static interference?
a) HF SITOR
b) Inmarsat
c) Digital Selective Calling on channel 70.
d) VHF ARQ

88. Which system is least likely to be affected by atmospheric disturbances?


a) NAVTEX
b) Inmarsat
c) MF NBDP
d) HF NBDP

89. Which system has the least effective radius of operation?


a) HF SITOR
b) MF NBDP
c) VHF DSC
d) NAVTEX

***Updated by: 2/M LINO A. ARRIESGADO


Date: OCT. 10, 2018
109
90. A vertical (whip) antenna has a radiation pattern best described by?
a) A figure eight
b) A cardioid
c) A circle
d) An ellipse

91. What advantage does a vertical whip have over a long wire?
a) It radiates more signal fore and aft.
b) It radiates equally well in all directions.
c) It radiates a strong signal vertically.
d) None of the above

92. Which of the following statements about a VHF vertical antenna is true?
a) The longer a VHF antenna the greater the signal gain.
b) The radiation pattern is a cardioid.
c) Maximum radiation is directly overhead.
d) The radiation pattern is a figure eight.

93. Which of the following maintenance functions is not the responsibility of the
GMDSS Radio Operator?
a) Visual inspection of equipment, including the antenna and associated components.
b) Perform on-the-air verification checks.
c) Perform scheduled testing of the battery's charged condition.
d) Aligning the power output stage for maximum power.

94. Equipment for radiotelephony use in survival craft stations under GMDSS must have what
characteristic(s)?
a) Permanently-affixed antenna, watertight, power 1W or 25W.
b) Watertight, power a minimum of 1W, operation on CH-16, Ch-13 & Ch-70.
c) Operation on Ch-16, watertight, permanently-affixed antenna.
d) Operation on Ch-16, Ch-13 & Ch-70, power 1W, permanently-affixed antenna.

95. Which statement is NOT true regarding the requirements of survival craft portable two-way VHF
radiotelephone equipment?
a) Operation on Ch-16
b) Antenna must be permanently-affixed.
c) Simplex (single frequency) voice communications only.
d) Effective radiated power should be a minimum of 2.0 Watts.

***Updated by: 2/M LINO A. ARRIESGADO


Date: OCT. 10, 2018
110
96. Which statement is NOT true regarding the requirements of survival craft portable two-way VHF
radiotelephone equipment?
a) Watertight to a depth of 1 meter for 5 minutes.
b) Operates simplex on Ch-70 and at least one other channel.
c) Effective radiated power should be a minimum of 0.25 Watts.
d) The antenna is fixed and non-removable.

97. How is mutual interference on 518 kHz among NAVTEX stations avoided?
a) Transmissions scheduled on a time-sharing basis, power limited and station assignment codes
are geographically separated.
b) All stations transmit at the same time but stations are limited to daytime operation only to
reduce the radius of propagation.
c) Transmitter power is limited, station assignment codes are not shared by other NAVAREAS
and stations alternate between daytime and nighttime operations.
d) Station codes are not shared by other NAVAREAS, transmissions scheduled on a time-
sharing basis and power is limited.

98. Describing VHF transmissions as "line of sight" does not mean:?


a) Vessel antenna height will not affect the radius of propagation.
b) The normal transmission range to a coast station is approximately is 10 NM.
c) Coast station antenna height has no effect on the radius of transmission.
d) VHF communications are effective only with nearby stations within visual range of the
bridge.

99. The effectiveness of VHF communications is maximized by:?


a) Appropriate setting of the transmitter power, selecting an appropriate channel & adjustment of
squelch for maximum receiver sensitivity.
b) The adjustment of squelch for maximum receiver sensitivity, setting transmitter power to 1W &
selecting an appropriate channel.
c) Selecting an appropriate channel, adjustment of squelch for minimum receiver sensitivity &
setting transmitter power to 1W.
d) Selecting an appropriate channel, adjustment of squelch for minimum receiver sensitivity, setting
transmitter power to 25W.

100. The VHF-DSC self-test function (if available) usually performs the following:?
a) Transmits a very weak DSC signal which is picked up by the Ch-70 receiver.
b) Transmits a weak voice signal which is picked up on Ch-16.
c) Makes a diagnostic test of the system without actually transmitting a signal.
d) Does a check sum of the memory chips.

***Updated by: 2/M LINO A. ARRIESGADO


Date: OCT. 10, 2018
111
101, Which statement best describes frequency modulation?
a) Both the amplitude and frequency are changed by the modulating signal.
b) The frequency is changed by the information signal and the amplitude remains unchanged.
c) Frequency modulation is subject to interference by atmospheric noise.
d) High level mixing of the final amplifier signal and the information signal.

102, How can rescue personnel detect that a SART is transmitting in the immediate vicinity?
a) The SART’s blips on the PPI will begin showing series of dots and eventually become
concentric circles
b) The SART’s blips on the PPI will begin showing rapid dashes and eventually become concentric
circles
c) Both a and b
d) The SART’s blips on the PPI will begin arcing and eventually become concentric circles

103, How is mutual interference among NAVTEX stations avoided?


a) Transmitting at maximum power output
b) Transmit MSI once every 24 hours
c) Transmitter power is limited to that necessarily for coverage of assigned area, transmissions by
stations in each NAVAREA are arranged in a time sharing basis
d) Transmit message type A, B and D only

104, What should an operator do to prevent interference?


a) Do not transmit
b) Adjust squelch
c) Monitor channel before transmitting and make transmission as brief as possible
d) Increase volume

105, When listening to the transmission of a very close station, what step must be observed to prevent
the strong signal from “smothering” the receiver?
a) Lower power output
b) Switch off speaker and use headset
c) Adjust volume control
d) Adjust squelch

106, Which of the following statements concerning a fixed RADIOTELEPHONE?


a) Messages detailing important navigational warnings
b) Fixed units operate in the routine mode with RF power level no greater than 1 watt
c) A serious medical situations involving a crewmember
d) Scenarios concerning the safety of navigation of important meteorological warnings.

***Updated by: 2/M LINO A. ARRIESGADO


Date: OCT. 10, 2018
112
107, What are the operation of emergency equipment & application of emergency procedures?
a) Distress signal
b) Routine signal
c) Safety Signal
d) Urgency Signal

108, In Morse Code signaling by Hand Flag or Arms, following are the transmitting signals to a
receiving station except for;?
a) To call up a station
b) A receiving station in answer
c) Completing the signal
d) Sending the distress message

109, A frequent maintenance is the basis for a reliable working condition of the battery, when
working on batteries, effective safety precautions must be taken except for;
a) Avoid over-discharging below 2.1 Volts for any cell
b) Ensure electrolyte level is maintained, but do not overfill, 1 cm above plates is adequate
c) Keep cells top clean and dry, check ventilation holes, tighten terminals and coat with Vaseline
d) Charge the batteries for more than specified.

110, Following are the safety measures to be taken during handling storage battery in substation
except for…?
a) Do not bring a flame inside the room
b) Do not allow the battery to stand idle for long time
c) Do not generate any spark inside the room
d) Wear plash proof googles, rubber gloves, while workinh with the battery

111, A vertical (whip) antenna has a radiation pattern best described by?
a) A figure eight
b) A cardioid
c) A circle
d) An ellipse

112, What advantage does a vertical whip have over a long wire?
a) It radiates more signal fore and aft.
b) It radiates equally well in all directions.
c) It radiates a strong signal vertically.
d) None of the above

113, Which of the following statements about a VHF vertical antenna is true?
a) The longer a VHF antenna the greater the signal gain.
b) The radiation pattern is a cardioid.
c) Maximum radiation is directly overhead.
d) The radiation pattern is a figure eight.
***Updated by: 2/M LINO A. ARRIESGADO
Date: OCT. 10, 2018
113
114, Which of the following maintenance functions is not the responsibility of the
GMDSS Radio Operator?
a) Visual inspection of equipment, including the antenna and associated components.
b) Perform on-the-air verification checks.
c) Perform scheduled testing of the battery's charged condition.
d) Aligning the power output stage for maximum power.

115, The lead acid battery SPECIFIC GRAVITY should be tested by means of hydrometer at least:
a) Once a day
b) once a week
c) once a month
d) once a year

116, The connection and condition of the reserve batteries should be checked:
a) Once a month
b) once a year
c) once every three months
d) once a week

117, When two 12-volt batteries with an individual capacity of 100 Ampere-hours (Ah) are coupled in
series, the result will be a battery with a total voltage and capacity of…
a) 12-volt and 100 Ah respectively
b) 12-volt and 200 Ah respectively
c) 24-volt and 100 Ah respectively
d) 24-volt and 200 Ah respectively

118, When measured directly above given point, solar radiation in the ionosphere is lowest or lesser
minimum during what time?
a) At night
b) At sunlight
c) At Noon
d) Sunrise or Night

119, When measured directly above given point, solar radiation in the ionosphere is minimum during
what time?
a) At sunlight
b) At night
c) At Noon
d) At sunset

***Updated by: 2/M LINO A. ARRIESGADO


Date: OCT. 10, 2018
114
120, When measured directly above given point, solar radiation in the ionosphere is the greatest or
maximum during what time?
a) At night
b) At sunlight
c) At sunrise
d) At Noon

***Updated by: 2/M LINO A. ARRIESGADO


Date: OCT. 10, 2018
115
GOC FOR GMDSS REVIEWER
COMPETENCE 1 Transmit and Receive information using GMDSS subsystem and equipment
and fulfilling the functional requirements of GMDSS (GOC)
GMDSS FUNDAMENTAL CONCEPT (ADDITIONAL REVIEWER)

1. What is the fundamental concept of the GMDSS?


a) It is intended to automate and improve emergency communications in the maritime industry.
b) It is intended to automate and improve existing digital selective calling procedures and techniques.
c) It is intended to provide more effective but lower cost commercial communications.
d) It is intended to provide compulsory vessels with a collision avoidance system when they are
operating in waters that are also occupied by non-compulsory vessels.

2.The primary purpose of the GMDSS is?


a) Allow more effective control of SAR situations by vessels.
b) Provide additional shipboard systems for more effective company communications.
c) Automate and improve emergency communications for the world's shipping industry.
d) Effective and inexpensive communications.

3.What is the basic concept of GMDSS?


a) Search and rescue authorities ashore can be alerted to a distress situation.
b) Shipping in the immediate vicinity of a ship in distress will be rapidly alerted.
c) Shoreside authorities and vessels can assist in a coordinated SAR operation with minimum delay.
d) All of these.

4.GMDSS is primarily a system based on?


a) Ship-to-ship distress communications using MF or HF radiotelephony.
b) VHF digital selective calling from ship to shore.
c) Distress, urgency and safety communications carried out by the use of narrow-band direct printing
telegraphy.
d) The linking of search and rescue authorities ashore with shipping in the immediate vicinity of a ship
in distress or in need of assistance.

5.What is the responsibility of vessels under GMDSS?


a) Vessels over 300 gross tons may be required to render assistance if such assistance does not
adversely affect their port schedule.
b) Only that vessel, regardless of size, closest to a vessel in distress, is required to render assistance.
c) Every ship is able to perform those communications functions that are essential for the safety of the
ship itself and of other ships.
d) Vessels operating under GMDSS, outside of areas effectively serviced by shoreside authorities,
operating in sea areas A2, and A4 may be required to render assistance in distress situations.

***Updated by: 2/M LINO A. ARRIESGADO


Date: OCT. 10, 2018
116
6. GMDSS is required for which of the following?
a) All vessels capable of international voyages.
b) Vessels operating outside of the range of VHF coastal radio stations.
c) SOLAS Convention ships of 300 gross tonnage or more.
d) Coastal vessels of less than 300 gross tons.

7.What equipment is associated with the land or terrestrial systems?


a) EPIRB.
b) VHF-MF-HF.
c) Inmarsat-C.
d) GPS.

8.What equipment is associated with the space systems?


a) VHF-MF-HF.
b) Inmarsat-C.
c) Navtex.
d) SART.

9.Which satellite system promulgate Maritime Safety Information?


a) AMVER.
b) Inmarsat-C SafetyNET.
c) NAVTEX.
d) Inmarsat-M SES.

10.The MMSI number plan is used in all of the following in GMDSS except?
a) VHFDSC, MFDSC, HFDSC and radio telex
b) Inmarsat-C
c) Inmarsat and COSPAS/SARSAT EPIRBs
d) SART

11.What means are used to prevent the reception of unwanted broadcasts by vessels utilizing the NAVTEX
system?
a) Operating the receiver only during daytime hours.
b) Programming the receiver to reject unwanted broadcasts.
c) Coordinating reception with published broadcast schedules.
d) Automatic receiver de-sensitization during night hours.

12.Which of the following systems is most likely to be subject to fading or static interference?
a) HF SITOR.
b) Inmarsat.
c) Digital Selective Calling on channel 70.
d) VHF ARQ.

***Updated by: 2/M LINO A. ARRIESGADO


Date: OCT. 10, 2018
117
13.Which system is most likely to be affected by atmospheric disturbances?
a) MF/HF radiotelephony.
b) VHF DSC.
c) Inmarsat.
d) SafetyNET

14.Which of the following systems is least likely to be subject to fading or static interference?
a) HF SITOR.
b) Inmarsat.
c) MF-HF DSC Controller.
d) VHF ARQ

15.Which system is least likely to be affected by atmospheric disturbances?


a) NAVTEX.
b) Inmarsat.
c) MF NBDP.
d) HF NBDP

16.Which system has the least effective radius of operation?


a) HF SITOR.
b) MF NBDP.
c) VHF DSC.
d) NAVTEX.

17.What is the term which refers to the supply of electrical energy required to supply radio installations on
every ship for the purpose of conducting distress and safety radio-communications in the event of failure
of the ship's main and emergency sources of electrical power?
a) Emergency power.
b) Reserve source of energy.
c) Ship's emergency diesel generator.
d) Ship's standby generator.

18.Which of the following is the primary frequency that is used exclusively for NAVTEX broadcasts
internationally?
e) 518 kHz
f) 2187.5 kHz
g) 4209.5 kHz
h) VHF channel 16 when the vessel is sailing in Sea Area A1, and 2187.5 kHz when in Sea Area
A2.

***Updated by: 2/M LINO A. ARRIESGADO


Date: OCT. 10, 2018
118
19.What is false about HF communication?
a) used for long range communication
b) can be reflected by the ionosphere
c) Not affected by time of day
d) can be used as alternative for Inmarsat communication

20.The transmission speed of a DSC call on the VHF band is…


a) 600 bauds
b) 1200 bauds
c) 1600 bauds
d) 2400 bauds

21.The NAVTEX message which ensures that the system is functioning normally is type?
a) A
b) B
c) Z
d) G

22. What is the fundamental concept of the GMDSS?


a) to repel piracy attacks
b) It is intended to automate and improve emergency communication in maritime industry
c) to increase participation in ship reporting system
d) to transport more cargoes effectively

23. DSC relays of Distress Alerts by vessels:


a) Should be done for all Distress Alerts.
b) Should be transmitted to ships involved in Distress Alert.
c) Should be avoided, however after repeated alerts should be relayed to a Coast Station nearest the
distress incident.
d) Are the best means to provide for a relay of Distress communications

24. What is the meaning of “Reserve Source of Energy?”


a) The supply of electrical energy sufficient to operate the radio installations for the purpose of
conducting distress and safety communications in the event of failure of the ship’s main and
emergency sources of electrical power.
b) High caloric value items for lifeboat, per SOLAS regulations.
c) Diesel fuel stored for the purpose of operating the powered survival craft for a period of equal to
or exceeding the U.S.C.G and SOLAS equipment.
d) None of these

***Updated by: 2/M LINO A. ARRIESGADO


Date: OCT. 10, 2018
119
25.A Faraway SART indicated by a series of dots on the radar display is approximately
a) 5 nautical miles
b) 4 nautical miles
c) 3 nautical miles
d) 2 nautical miles

26. What is the global maritime distress and safety system (GMDSS)?
a) A new system to increase participation in ship reporting system
b) An automated ship-to-shore distress alerting system using satellite and advanced terrestrial
communications systems
c) A new system to transport more cargoes effectively
d) A new system to repel piracy attacks

27.Which communication functions must all vessels be capable of performing under GMDSS as defined by
international maritime organization?
a) Ship to ship alerting
b) Ship to shore alerting
c) Shore to Ship alerting
d) Distress alerting to and from vessels, search and rescue coordination, on-scene communication,
signals for locating, maritime safety information, general and bridge-to-bridge communications

28.Which statement is not true regarding SART?


a) Operates in conjunction with a vessel S-Band radar
b) Once it has been placed in the “ON” position, a SART will begin transmitting immediately upon
detecting that it is the water
c) This is 9 GHZ transponder capable of being received by another vessels X-band navigational
radar system
d) Both a & b

29.The best way to test the MF-HF NBDP system is?


a) Make a radiotelephone call to a Coast Station.
b) Initiates an ARQ call to demonstrate that the transmitter and antenna are working.
c) Initiate an FEC call to demonstrate that the transmitter and antenna are working.
d) Initiate an ARQ call to a Coast Station and wait for the automatic exchange of answerbacks.

30.The type of batteries most commonly used on board ships as source of backup power is the?
a) lithium battery
b) acid battery
c) Lead battery
d) rechargeable battery

***Updated by: 2/M LINO A. ARRIESGADO


Date: OCT. 10, 2018
120
31.What is proper procedure to be followed upon receipt of a distress alert transmitted by use a digital
selective calling techniques?
a) Set watch on the radio telephone distress and safety frequency associated with distress and safety
calling frequency on which the DISTRESS alert was received
b) Send distress relay alert at once
c) Inform RCC immediately
d) Inform the Master of the vessel

32. A MF/HF DSC Distress call:


a) Will send the minimal necessary information using the "Distress Button" or "Distress Hot Key"
b) Contains all the information normally of interest in on-scene Distress communications.
c) Will send a more detailed Distress format if time permits and operator data entries are correctly
performed.
d) Both a) & c) are true.

33.Repetition of a DSC Distress call is normally automatic if not acknowledged after a delay of:
a) 1 - 2 minutes.
b) 2- 5 minutes.
c) 3.5 - 4.5 minutes.
d) Not at all.

34.DSC relays of Distress Alerts by vessels:


a) Should be avoided, however after repeated alerts, should be relayed to a Coast Station nearest the
Distress incident.
b) Should be done for all Distress Alerts.
c) Should be transmitted to ships involved in Distress traffic.
d) Are the best means to provide for a relay of Distress communications.

35.The frequency band used as uplink frequency from CES to the satellite is the:
a) 1525.0 - 1545.0 MHZ (1.5GHZ )
b) 6425.0 - 6443.0 MHZ ( 6GHZ )
c) 3600.0 - 3623.0 MHZ ( 4 GHZ )
d) 1626.5 - 1646.5 MHZ (1.6 GHZ)

36.The frequency band used as downlink frequency from Satellite to the CES is the:
a) 1525.0 - 1545.0 MHZ (1.5GHZ )
b) 6425.0 - 6443.0 MHZ ( 6GHZ )
c) 3600.0 - 3623.0 MHZ ( 4 GHZ )
d) 1626.5 - 1646.5 MHZ (1.6 GHZ)

***Updated by: 2/M LINO A. ARRIESGADO


Date: OCT. 10, 2018
121

S-ar putea să vă placă și